Задачи на составление уравнений как решать: Решение задач на составление уравнений — урок. Математика, 6 класс.

Содержание

Схема решения текстовых задач

Текстовые задачи на составление уравнений изучают в 8, 9 классе. Сложные или простые задачи способствуют подготовке школьников к олимпиаде, тестам, вступительным экзаменам.
Среди задач рассмотренных в статье есть задачи на движение, на возраст, о треугольнике, совместную работу.
Цель таких задач — научить Вас составлять уравнения к задаче и решать их.

Схема решения задачи на составление уравнений

Перед решением задач необходимо провести анализ, который выполняется по схеме:

  • Определение величин указанных в условии задачи.
  • Установление зависимости между указанными величинами.
  • Определение главного вопросу задачи.
  • Обоснование выбора неизвестной величины (или величин).
  • Выражение других величин задачи через неизвестную.
  • Составление уравнения к задаче.
  • Решение уравнений.
  • Выяснение удовлетворяют ли найденные корни уравнения условие задачи.
  • Дать ответ на главный вопрос задачи.

Для приобретения необходимого опыта нужно разобрать много задач, изучить алгоритмы составления уравнений, схемы возведения уравнений к простому виду. Для этого рассмотрим простые задачи и по мере изучения темы «Текстовые задачи на составление уравнений» разберем задачи от простых до сложных.

Решения задач на составление уравнений

Задача 1. Турист прошел 20% всего пути. Осталось пройти на 36 км больше чем прошел. Какова длина пути (в км) ?
Решение: В подобных задачах можете выполнять дополнительное графическое построение для понимания условия задачи. Прошел 20% означает, что это 20/100 = 0,2 от всего пути. Осталось пройти на 36 км больше, чем прошел.
Итак весь путь равный
0,2+0,2+36 км=1.
Отсюда (1-0,2-0,2)=0,6 или 60% отвечает за 36 км.
Составляем пропорцию
36 км – 60%
x – 100%.
Перекрестным умножением определяем весь путь
x=36*100/60=36/0,6=60 (км).
Ответ: Длина пути 60 км.

Задача 2. Турист пришел 1/5 пути. Осталось пройти на 18 км больше чем он прошел. Какова длина пути (в км)?
Решение: Задача на определение пути по схеме вычислений идентична предыдущей задаче.
По условию туристу осталось пройти 1/5 пути +18 км.
Устанавливаем, какая доля пути равна 18 км
1-1/5-1/5=3/5.
Поделив на нее получим длину всего пути
18:3/5=18*5/3=30(км)
Ответ: длина пути 30 км.

Задача 3. Турист прошел 0,3 пути. Осталось пройти на 30 км больше чем он прошел. Какова длина пути (в км)?
Решение: Распишем задачу в объяснениях.
Пусть х — весь путь
0,3*х – прошел
0,3*х+30 км осталось
Вычислим сколько занимает 30 км от всего пути
х-0,3*х-0,3*х=0,4*х.
Из уравнения находим искомое расстояние
0,4*х=18; х=18:0,4=45(км)
Ответ: Длина пути 45 км.

Задача 4. Мать старше дочери в 4 раза. Вместе им 40 лет. Сколько лет дочери?
Решение: Такого рода задач на составление уравнений немало. Алгоритм вычислений следующий.
Пусть дочери х лет, тогда матери 4 * х лет.
По условию составляем уравнение
х+4*х=5*х;
5*х=40.
Отсюда находим возраст девочки
х=40/5=8 (лет)
Ответ: Дочери 8 лет.

Задача 5. Мать старше дочери на 24 года. Вместе им 40 лет. Сколько лет матери?
Решение: Обозначим через Х возраст дочери. Тогда (Х + 24) — возраст матери.
Далее составим уравнение из условия, что сумма лет равна 40.
Х+Х+24=40;
2*Х=40-24=16;
Х=16:2=8 (лет).
Найдем возраст матери
Х+24=8+24=32 (года)
Ответ: Матери 32 года.

Задача 6. Цену товара увеличили на 53%. Во сколько раз стал дороже товар?
Решение: Начальная цена товара составляет 100%. Увеличили на 53% означает
100%+53%=153%.
Далее вычисляем отношение образованной цены к начальной
153%/100%=1,53(раза)
Ответ: Товар стал дороже в1,53 раза.

Задача 7. Отец старше сына в 2 раза. Сколько лет сыну если отец старше на 18 лет?
Решение: Пусть сыну Х лет. Тогда отцу по условию 2х лет.
Старший на 18 лет означает, что разница лет равна 18.
В наших обозначениях условие равносильно уравнению
2*Х-Х=Х=18 лет.
Ответ: сыну 18 лет.

Задача 8. Отец старше сына в 5 раз. Сколько лет отцу если он старше сына на 20 лет?
Решение: Пусть сыну Х лет отцу
Х*5=5*Х лет
Из-за разницы составляем уравнения возраста
5*Х-Х=20;
4*Х=20.
Находим возраст сына
Х=20:4=5 лет
дальше возраст отца
5*Х=5*5=25 (лет).
Ответ: Отцу 25 лет.

Задача 9. Острые углы прямоугольного треугольника относятся как 2: 1. Сколько градусов имеет меньший острый угол?
Решение: Здесь нужно знать что сумма углов треугольника равна 180 градусов.
Один из углов прямой, поэтому на два других приходится
180-90=90 градусов.
Обозначим меньший угол через Х, тогда другой 2Х.
составим уравнение
2*Х+Х=900;
3*Х=900;
Х=900/3=300
Ответ: Острый угол треугольника имеет 300.

Задача 10. Стороны треугольника относятся как 2: 3: 4. Вычислить длину большей стороны если его периметр равен 180.
Решение: Согласно условию обозначим стороны треугольника — 2*Х; 3*Х; 4*Х.
Далее составляем уравнение относительно неизвестной и решаем его
2*Х+3*Х+4*Х=180;
9*Х=180;
Х=180/9=20.
Находим большую сторону треугольника
4*Х=4*20=80 (единиц).
Ответ: Длина стороны 80.

Задача 11. Углы треугольника относятся как 1: 3: 6. Сколько градусов имеет средний угол?
Решение: Вводим обозначения углов согласно их пропорции Х: 3*Х: 6*Х.
Составляем уравнение
Х+3*Х+6*Х=1800;
10*Х=1800;
Х=1800/10=180.
Находим меру среднего угла
3*Х=18*3=540;
Ответ: Искомый угол треугольника равен 54 градуса.

Задача 12. За два дня обработали 160 га пшеницы, причем в первый день обработали на 36 га больше чем второго. Сколько гектаров обработали второго дня?
Решение: Обозначим Х — площадь, которую обработали пшеницы второго дня.
По условию Х + 36 га — в первый день.
составляем уравнение
Х+Х+36=160;
2*Х=160-36=124;
Х=124/2=62 (га).
Ответ: Во второй день обработали 62 га пшеницы.

Задача 13. За два дня обработали 140 га пшеницы, причем в первый день обработали на 30 га больше чем второго. Сколько гектаров обработали первого дня?
Решение: Обозначаем Х га — обработали второго дня Х + 30 га — в первый день.
записываем уравнение
Х+Х+30= 140(га;)
2*Х=140-30=110 (га)
Х=110/2=55(га).
Найдем площадь обработки первого дня
55+30=85 (га).
Ответ: В первый день обработали 85 га пшеницы.

Задача 14. Два рабочие изготовили вместе 84 детали, работая 7 дней. Сколько деталей в день изготавливал первый рабочий если второй изготавливал за день на 2 детали меньше?
Решение:Обозначим через Х количество деталей, которое производит первый рабочий. Тогда второй изготовляет — Х-2 деталей.
Составляем уравнение
(Х+Х-2)*7=84.
Думаю здесь Вам все понятно, мы умножили производительности рабочих за день на количество дней.
(2*Х-2)*7=84;
2*Х-2=84/7=12;
2*Х=12+2=14;
Х=14/2=7(деталей).
Ответ: Первый рабочий производит 7 деталей.

Задача 15. Сумма двух чисел равна 12, а их разность равна 4. Найти больше из чисел.
Решение: Обозначим числа через а и b. По условию задачи составляем уравнение.
а+b=12;
а-b=4.
Имеем систему двух уравнений с двумя неизвестными.
Добавим к 1 уравнение 2, таким образом обнулим переменную b
2а=12+4=16;
а=16/2=8;
b=12-a=12-8=4.
Ответ: большее число равно 8.

Посмотреть похожие материалы:

  • Решение текстовых задач. 8 класс
  • Задания на составление уравнений
  • Решение задач составлением уравнений
  • Составление уравнений к текстовым задачам
  • Решение примеров на составление уравнений
  • Решения к текстовым задачам на составление уравнений

Решение задач с помощью уравнений

Разделы: Математика

Класс: 6


Предмет

Математика

Класс

6-й

Тема и номер урока в теме

Решение задач с помощью уравнений (второй урок из пяти в данной теме).

Базовый учебник

А.Г.Мерзляк. Математика: 6 класс: учебник. – М.: Вентана-Граф, 2019.

Цель урока: закрепление навыков решения уравнений и задач на составление уравнений.

Задачи: Развитие математической речи.

Ход урока

1. Организационный момент

Ребята, на прошлом уроке мы с вами научились решать задачи на составление уравнений. Как вы думаете, чем сегодня мы будем заниматься на уроке? А как вы думаете, какова цель сегодняшнего урока? Конечно же, сегодня мы с вами продолжим решать задачи на составление уравнений.

2. Актуализация знаний

Решите уравнения (Ответы открываются после решения).

2(х + 8) = 3х +3,

Ответ: х = 13

5х + 18 = 2(х + 6) + 2х + 27,

Ответ: х = 21

3х – 20 = 2(х – 1),

Ответ: х = 18

3(2a + 3) – 4a = 3(a – 1),

Ответ: a = 12

4(2у – 5) – 13 = 3у – 3,

Ответ: у = 6

3. Задания для фронтальной работы

Назовите этапы при решении задач.

  1. Выбери соответствующее выражение данному утверждению (даны несколько выражений).
  2. В одном бидоне x литров, а в другом – y литров молока. На сколько литров во втором бидоне меньше молока, чем в первом?
  3. Пусть x – число телевизоров на втором складе. На первом складе было в 3 раза больше телевизоров, чем на втором. После того как с первого склада взяли 11 телевизоров, а на второй привезли 17, телевизоров на обоих складах стало поровну. Какое из уравнений соответствует условию задачи? (даны варианты уравнений).
  4. Составь уравнение к задаче, начало решения которой показано в таблице:

 

1-й кабинет

2-й кабинет

Было стульев

x

2x

Осталось стульев

x−8

2x−34

Известно, что число стульев, оставшихся в кабинетах, было одинаковым. Определи число стульев, которые были в 1-м кабинете.

4. Составьте уравнение к задачам

1. На одной автостоянке было в 4 раза меньше машин, чем на другой. Когда со второй стоянки на первую перевели 72 автомобиля, машин на стоянках стало поровну. Сколько машин было на каждой стоянке первоначально?

2. В строительстве дороги принимали участие две бригады, причём число рабочих первой бригады составило 54 % числа всех рабочих двух бригад вместе. Сколько рабочих в каждой бригаде, если в первой бригаде на 8 человек больше, чем во второй?

Мы составляли уравнения к задачам. Теперь решим задачу вместе.

3. Длина прямоугольника на 2,2 см больше его ширины. Найдите стороны прямоугольника, если его периметр равен 18 см. (1 человек решает у доски, остальные – в тетрадях).

Ответ: 3, 4 см – ширина, 5, 6 см – длина.

5. Самостоятельная работа со взаимопроверкой

Ученики в течение 5-7 минут выполняют решение одной из задач в тетрадях, затем меняются тетрадями с соседом. На доске появляется решение задач. Ребята проверяют друг у друга решение, исправляя решение карандашом.

1. Мама на день рождения сына в школу купила конфеты, печенье и зефир. Зефир на 20 р., а конфеты на 50 р. стоят дороже, чем печенье. Определите цену за кг каждого продукта, если за 2 кг конфет, 3 кг печенья и 1 кг зефира было уплачено 1320 р.?

2.Петя в булочной купил кондитерские изделия: кекс, рогалик, булочку и пирожное. Кекс на 5 р., а пирожное в 3 раза дороже булочки. Определите стоимость каждого изделия, если рогалик в 2 раза дороже кекса и он равен стоимости пирожного?

6. Рефлексия

Какой навык закрепили сегодня на уроке? (решение задач при помощи уравнений).

  • Сколько существует этапов в решении данных задач? (три). Что нового узнали на уроке?
  • Можно ли использовать уравнения в повседневной жизни? Какое из заданий вам больше всего понравилось?

7. Подведение итогов и запись домашнего задания

Два задания из учебника (для всех), одну задачу составить самому и решить её (по желанию). Объявление оценок.

задачи на составление уравнений 5 класс | Учебно-методический материал по алгебре (5 класс) на тему:

Опубликовано 09.06.2013 — 19:04 — БУРДЫГИНА ИРИНА НИКОЛАЕВНА

подборка задач на закрепление навыков решения задач на составление уравнений для 5 класса

Скачать:

Реклама

Подтяните оценки и знания с репетитором Учи.ру

За лето ребенок растерял знания и нахватал плохих оценок? Не беда! Опытные педагоги помогут вспомнить забытое и лучше понять школьную программу. Переходите на сайт и записывайтесь на бесплатный вводный урок с репетитором.

Вводный урок бесплатно, онлайн, 30 минут

Записаться >


Предварительный просмотр:

ЗАДАЧИ НА СОСТАВЛЕНИЕ УРАВНЕНИЙ (5 КЛАСС)

  1. Света задумала число, умножила его на 4 и к произведению прибавила 8. В результате она получила 60. Какое число задумала Света?
  2. Собрали несколько килограммов свежей вишни. После того, как из 7 кг сварили варенье, а затем собрали ещё 5 кг, то свежей вишни стало 10 кг. Сколько вишни собрали изначально?
  3. В одной корзине в 6 раз меньше яблок, чем в другой. Сколько яблок в каждой корзине, если в двух корзинах 98 яблок?
  4. В трёх автобусах 188 пассажиров, причём в первом автобусе на 9 пассажиров больше, чем во втором, и на 8 меньше, чем в третьем. Сколько пассажиров в каждом автобусе?
  5. В двух залах кинотеатра 460 мест. Сколько мест в большом зале, если известно, что в нём в 3 раза больше мест, чем в малом зале?
  6. В школе 900 учащихся. Сколько учащихся в начальных, средних и старших классах, если в начальных классах их в 3 раза больше, чем в старших, и в 2 раза меньше, чем в средних?
  7. Площадь кухни в 3 раза меньше площади комнаты, поэтому для ремонта пола кухни потребовалось на 24 м2 линолеума меньше, чем для комнаты. Какова площадь кухни?
  8. Одна сторона прямоугольника в 4 раза меньше другой. Чему равны длина и ширина прямоугольника, если его периметр равен 70 см?
  9. На пруду плавали белые и серые утки, причём серых было в 3 раза больше, чем белых. После того, как на пруд прилетели 5 лебедей, то птиц всего оказалось 29. Сколько серых уток плавало на пруду?
  10.  В 5 «Б» классе из 27 учащихся «3» получили за контрольную по математике в 6 раз меньше человек, чем «4» и в 2 раза меньше, чем «5». Сколько учащихся получили «3», «4» и «5» за контрольную работу?

                                                                                                                   С любовью Бурдыгина И.Н.

ЗАДАЧИ НА СОСТАВЛЕНИЕ УРАВНЕНИЙ (5 КЛАСС)

  1. Света задумала число, умножила его на 4 и к произведению прибавила 8. В результате она получила 60. Какое число задумала Света?
  2. Собрали несколько килограммов свежей вишни. После того, как из 7 кг сварили варенье, а затем собрали ещё 5 кг, то свежей вишни стало 10 кг. Сколько вишни собрали изначально?
  3. В одной корзине в 6 раз меньше яблок, чем в другой. Сколько яблок в каждой корзине, если в двух корзинах 98 яблок?
  4. В трёх автобусах 188 пассажиров, причём в первом автобусе на 9 пассажиров больше, чем во втором, и на 8 меньше, чем в третьем. Сколько пассажиров в каждом автобусе?
  5. В двух залах кинотеатра 460 мест. Сколько мест в большом зале, если известно, что в нём в 3 раза больше мест, чем в малом зале?
  6. В школе 900 учащихся. Сколько учащихся в начальных, средних и старших классах, если в начальных классах их в 3 раза больше, чем в старших, и в 2 раза меньше, чем в средних?
  7. Площадь кухни в 3 раза меньше площади комнаты, поэтому для ремонта пола кухни потребовалось на 24 м2 линолеума меньше, чем для комнаты. Какова площадь кухни?
  8. Одна сторона прямоугольника в 4 раза меньше другой. Чему равны длина и ширина прямоугольника, если его периметр равен 70 см?
  9. На пруду плавали белые и серые утки, причём серых было в 3 раза больше, чем белых. После того, как на пруд прилетели 5 лебедей, то птиц всего оказалось 29. Сколько серых уток плавало на пруду?
  10.  В 5 «Б» классе из 27 учащихся «3» получили за контрольную по математике в 6 раз меньше человек, чем «4» и в 2 раза меньше, чем «5». Сколько учащихся получили «3», «4» и «5» за контрольную работу?

                                                                                                                   С любовью Бурдыгина И.Н.


По теме: методические разработки, презентации и конспекты

Урок по теме «Уравнения. Решение задач на составление уравнений»

презентация урока…

Элективный курс по математике в 5 классе «Решение уравнений. Задачи на составление уравнений»

Курс строится на индуктивной основе с привлечением элементов дедуктивных рассуждений. Теоретический материал курса излагается на наглядно-интуитивном уровне, математические методы и законы формулируют…

Разработка урока по теме: Решение задач на составление уравнений 6 класс

План-конспект  и презентация к уроку математики в 6 классе по теме: «Решение задач на составление уравнений» с включением элементов ФГОС. ..

7 класс, алгебра, обучающая самостоятельная работа по алгебре по теме: «Решение уравнений и задач на составление уравнений»

В самостоятельной работе даётся образец  типичного уравнения с пошаговым его решением. Далее предлагается решить четыре уравнения, постепенно, усложняя их.Так же разобран пример решения типичной …

5 класс. Математика.Проект урока и презентация к уроку по теме: решение уравнений и несложных задач на составление уравнений, урок в коррекционном классе 7-го вида

Урок может быть использован как для класса коррекции, так и для общеобразовательного класса.Урок проходит в виде соревнования по рядам, при этом дети сидят на своих местах, как они привыкли. Формы раб…

5 класс. Математика.Проект урока и презентация к уроку по теме: решение уравнений и несложных задач на составление уравнений, урок в коррекционном классе 7-го вида

Урок проходит в игровой форме, в виде соревнования по рядам. Урок состоит из 10 этапов. В процессе урока учащиеся участвуют в 7 конкурсах.

5 класс. Математика.Проект урока и презентация к уроку по теме: решение уравнений и несложных задач на составление уравнений, урок в коррекционном классе 7-го вида

Урок может быть использован, как для проведения в коррекционном классе, так и для общеобразовательного класса….


Поделиться:

 

Уравнения. Решение задач с помощью уравнений 5 класс онлайн-подготовка на Ростелеком Лицей

Введение

 

Для начала дадим краткое определение уравнению. Разберем, в каких областях математики оно встречается. Слово «уравнение» производное от слов «уравнивать», «равняться». Также оно является однокоренным со словом «равенство», которое нам уже встречались неоднократно. Приведем примеры равенств:

 

Важно вспомнить, что равенства бывают верные и неверные. Рассмотрим пример неверного равенства: . Отметим, что в левой и правой частях равенств, приведенных в примерах, написаны только числовые выражения. Мы знаем, что есть еще и буквенные выражения. Например, .

Возникает вопрос, откуда может взяться такое выражение и зачем приравнивать такое выражение к какому-нибудь числу (). В таком равенстве мы уже не можем проверить, верное оно или нет. Давайте разберем на примере, откуда такое равенство может взяться, зачем нам оно нужно и что за  в нем стоит.

 

Решение задач

 

 

Дано: нам нужно взвесить арбуз. Мы знаем, что если на одну чашу весов положить арбуз и гирю массой  килограмма, а на другую гирю массой  килограммов, то весы уравновесятся. Найдите массу арбуза.

 

Путем нехитрых вычислений мы определяем, что масса арбуза  кг. Может возникнуть вопрос, почему мы взвешивали арбуз именно так, ведь можно было просто уравновесить весы, поставив на другую чашу гирю массой  кг. Ответ простой, ведь может быть и так, что в нашем распоряжении есть только гири по  и  кг.

Давайте попробуем решить данную задачу через составление уравнения.

Решение: пусть  – вес арбуза, тогда на чаше весов с арбузом будет вес . По условию мы знаем, что на противоположной чаше находится  кг и весы уравновешены. Можем составить уравнение.

Ответ:  кг.

Теперь становится понятно, в каком случае мы можем вводить в равенства переменные.

Уравнением называется равенство двух выражений, в которых есть буквенная переменная.

Выходит, что уравнения нужны для того, чтобы находить значение буквенной переменной, которая обращает уравнение в верное равенство. Это приводит нас к определению того, что же означает решить уравнение.

Решить уравнение – значит найти все значения буквенной переменной, при подстановке которых уравнение обращается в верное равенство (или доказать, что таких значений нет).

Важно отметить, что уравнение может иметь больше одного решения, но с такими уравнениями мы познакомимся позже. В некоторых уравнениях вам может встретиться несколько переменных, но решить такое уравнение вам пока будет сложно, так как найти все возможные корни достаточно затруднительно. Пример такого уравнения: .

Можно сказать, что уравнение чаще всего составляют при решении каких-то практических задач. Таким образом, составив уравнение, мы можем решить его и найти неизвестную величину.

 

Решение уравнений путем переноса слагаемых

 

 

Иногда уравнение можно решить подбором, но легче всего пользоваться несколькими правилами, которые упростят для вас вычисления. Разберемся с ними на примере.

 

Дано: через  лет Коле исполнится . Сколько лет Коле в данный момент?

Решение: пусть  – возраст Коли (на данный момент в годах), тогда через  лет ему будет . Из условия задачи известно, что ему через  лет будет  год. Составим и решим уравнение: .

Стоит отметить, что уравнение не меняется, если применить любое действия к обеим его частям. В данном случае отнимем с каждой стороны по : .

Ответ: Коле сейчас  лет.

Действие, которое мы применили для решения уравнения, называется переносом слагаемого из одной части уравнения в другую. Важно помнить, что при переносе выражения знак перед ним меняется на противоположный.

Рассмотрим еще один пример: . В этом уравнении нам нужно перенести тройку. Чтобы избавиться от нее в левой части уравнения, нужно прибавить три, соответственно, и к правой части прибавляем тройку:

Решим еще одну задачу.

Дано: Ксения задумала натуральное число, к этому числу она прибавила , после чего из суммы вычла задуманное число. Далее к полученному числу она прибавила  и в итоге получила . Какое число задумала Ксения?

Решение: пусть  – число, которое задумала Ксения, тогда мы можем составить уравнение с учетом преобразований задуманного числа.

Потренируем перенос, начнем с восьмерки:

В итоге мы пришли к верному числовому равенству, значит, оно верное для любого икса. Можно сделать вывод, что, какое бы число ни задумала Ксения, у нее все равно выйдет одиннадцать.

Ответ: Ксения могла задумать любое число.

Рассмотрим подобную задачу и решим ее составив уравнение.

Дано: Дмитрий задумал натуральное число, прибавил к нему , вычел из него , вычел задуманное число и получил . Какое число задумал Дмитрий?

Решение: пусть  – задуманное Дмитрием число, тогда можем составить уравнение.

В итоге мы получили неверное равенство, и это приводит нас к заключению, что решений это уравнение не имеет.

Значит, в условии задачи ошибка и  получить в результате указанных действий Дмитрий не мог.

 

Заключение

 

 

На этом уроке мы познакомились с понятием уравнения. Выяснили, что значит решить уравнение, познакомились с методами решения уравнений. Также мы выяснили, для чего нужны уравнения и как решать с их помощью задачи.

 

 

Список рекомендованной литературы

  1. Математика 5 класс. Виленкин Н. Я., Жохов В. И., Чесноков А. С., Шварцбурд С. И., 31-е изд., стер. — М: Мнемозина, 2013. — 280 с.
  2. Математика 5 класс. Ерина Т. М. Рабочая тетрадь к учебнику Виленкина Н. Я., М.: Экзамен, 2013. — 128 с.
  3. Математика 5 класс. Мерзляк А. Г., Полонский В. Б., Якир М. С., М.: Вентана — Граф, 2013.

 

Рекомендованные ссылки на ресурсы сети Интернет

  1. Интернет-портал «school-assistant.ru» (Источник)
  2. Интернет-портал «mat-zadachi.ru» (Источник)
  3. Интернет-портал «uroki.tv» (Источник)

 

Домашнее задание

1) Решите уравнения.

2) На правой чашке уравновешенных весов лежат дыня и гиря массой  кг, а на левой чашке – гиря массой  кг. Какова масса дыни?

3) Составьте и решите уравнение:

  1. Сумма удвоенного числа  и числа  равна .
  2. Разность чисел  и  в  раза меньше числа .
  3. Частное суммы чисел  и  и числа  равно .
  4. Сумма чисел  и  в  раза больше числа .
  5. Частное разности чисел  и  и числа  равно .
  6. Утроенная разность чисел  и  равна .

 

Глава 18 Задачи на составление уравнений . Сборник задач по математике с решениями для поступающих в вузы

При решении задач на составление уравнений основную трудность представляет перевод условия задачи с обычного языка на язык математических символов и уравнений. Наиболее ответственный этап этого процесса — выбор неизвестных. Нельзя шаблонно выбирать в качестве неизвестных величины, стоящие в вопросе задачи. Основное требование, которому должны отвечать выбранные неизвестные, состоит в том, чтобы с их помощью можно было прозрачно записать сформулированные в условии задачи соотношения.

Разберем в качестве примера следующую задачу.

Пример 1. Трое рабочих должны изготовить некоторое число деталей. Сначала к работе приступил первый, а через некоторое

время к нему присоединился второй. Когда 1/6 работы была выполнена, к работе приступил третий. Работу они закончили одновременно. Сколько времени работал первый рабочий, если каждый изготовил одинаковое число деталей, причем третий работал на 2 ч меньше второго? Известно, что первый и второй, работая вместе, могут изготовить требуемое число деталей на 9 ч раньше, чем третий, если бы он работал один.

Известно, что каждый рабочий изготовил одинаковое число деталей, т. е. выполнил треть всей работы. С другой стороны, нет никаких сведений о числе деталей, изготовленных кем-либо в какой-либо промежуток времени. Это означает, что речь идет о работе «вообще», о том, что каждый выполнял какую-то часть этой работы, а потому всю работу следует принять за единицу. Ту же мысль подтверждает и условие, в силу которого третий рабочий приступил к работе, когда

1/6 работы (обратите внимание: 1/6 всей работы, а не 45 или 27 деталей) была уже выполнена.

Из условия следует, что рабочие работают по-разному, другими словами, они изготовляют разное число деталей за одно и то же время. Поэтому нужно ввести в рассмотрение производительность каждого из них. Однако через x, у и z мы обозначим не число деталей, изготовляемых в час первым, вторым и третьим рабочими соответственно, а ту часть всей работы, которую каждый из них выполняет за это время.

После всего сказанного должно быть очевидным, что мы легко перепишем условие задачи в виде системы уравнений, если введем в рассмотрение еще три неизвестные:

t1, t2, t3 — время, затраченное соответственно первым, вторым и третьим рабочими. Так как каждый из них сделал за это время треть всей работы, то

t1x = t2у = t3z = ?. (1)

Мы получили три уравнения (их можно было написать в виде t1x = ?, t2у = ?, t3z = ?.  K ним нередко добавляют четвертое:

t1x + t2у + t2z = 1,

которое должно отражать то обстоятельство, что в итоге вся работа была выполнена. Однако это уравнение не содержит никакой самостоятельной информации: оно является следствием первых трех и получается в результате их сложения. Поэтому последнее уравнение, хотя и верно составлено, но бесполезно для решения задачи.

Так как первый и второй рабочие вместе выполняют всю работу за 1/x + y ч, а третьему на это потребуется 1/z ч, то еще одно условие задачи можно записать так:

1/x + y  + 9 = 1/z.     (2)

Составим теперь уравнение, отражающее тот факт, что третий рабочий приступил к работе, когда ее 1/6 была выполнена. Другими словами, когда первый проработал t1 ? t3 ч, а второй t2 ? t3 ч, они сделали 1/6 всей работы:

x(t1 ? t3) + у(t2 ? t3) =

1/6.    (3)

Добавляя к этим пяти уравнениям шестое:

t2 ? t3 = 2,      (4)

мы можем приступить к решению полученной системы уравнений.

Решая систему уравнений, как правило, следует держать в поле зрения два обстоятельства. Во-первых, систему уравнений нужно воспринимать в целом, так, как вы воспринимали бы ее, решая вне связи с задачей. Это позволит найти более рациональный ключ к ее решению. Во-вторых, нельзя упустить из виду те неизвестные (или комбинации неизвестных), которые позволят ответить на вопрос задачи. Благодаря этому можно обойтись без излишних вычислений.

В нашем примере второе обстоятельство должно побудить нас использовать уравнение (4) для упрощения уравнения (3), в результате чего из (3) будет исключено неизвестное t2, которое нас не интересует. Однако после замены t2 ? t3 на 2 уравнение (3) потеряет симметрию относительно t1x и t2у, что затруднит использование уравнений (1). Если же в уравнении (3) раскрыть скобки и вспомнить, что xt1 = ? и уt2 = ?, то получим уравнение

t3(x + у) = ?.

С его помощью можно выразить x + у через t3, а из уравнения zt3 = ? можно выразить через t3 и неизвестное z. Подставляя эти выражения в (2), получим

2t3 + 9 = 3t3,

откуда

t3 = 9.

Дальнейшее решение системы не представляет труда. Находим последовательно: t2 = 11, z = 1/27, у = 1/33. Из уравнения (2) определяем x = 5/198 и t1 = 1/3x= 66/5. Итак, первый рабочий работал 13 ч 12 мин.

Эту же задачу можно было бы решить с помощью меньшего числа неизвестных, если ввести в рассмотрение, помимо величин x, у и z, имеющих прежний смысл, величину t, обозначающую время, в течение которого рабочие работали вместе, т. е. время работы третьего рабочего. Это привело бы нас к системе:

t(x + у + z) = 5/6     (1?)

(за время t рабочие сделали вместе 5/6  всей работы),

tz = (t + 2)у = ?     (2?)

(за время t третий рабочий сделал треть всей работы, а второму на это потребовалось на 2 ч больше),

1/x + y + 9 = 1/z     (3?)

(первый и второй рабочие выполняют всю работу на 9 ч быстрее, чем третий, работая один).

Поскольку tz = ?, то из (1?) найдем 

x + y = 1/2t

Вместе с z = 1/3t подставим в (3?). Получим

t = 9.

Как и прежде, найдем последовательно z, у и x. На вопрос задачи можно ответить, вспомнив, что первый рабочий работал столько, чтобы успеть сделать ? всей работы, т. е. 1/3x.

Конечно, второе решение выглядит более изящно, чем первое. Однако признать его лучшим трудно, поскольку за те простые уравнения, от которых мы отказались, пришлось уплатить некоторым усложнением логики.

А теперь приведем арифметическое решение этой задачи — решение, в котором удается обойтись вообще без составления уравнений.

Так как рабочие совместно выполнили 1 ? 1/6 = 5/6 всей работы, причем третий сделал ?, то на долю первого и второго осталось 5/6 ? ? = ? всей работы. Следовательно, если бы первый и второй успели выполнить всю работу, то третий за то же самое время сделал бы ?; ему останется 1 ? ? = ? , на что ему потребовалось бы в силу последнего условия задачи 9 ч.

Так как каждый рабочий сделал одинаковое количество деталей, т. е. ? всей работы, то третий работал ровно 9 ч. Тогда второй работал 9 + 2 = 11 ч. Так как он тоже сделал ? всей работы, то его производительность равна 1/33 всей работы в час. Мы знаем, что первый и второй тратят на ? всей работы столько же, сколько третий на ?, т. е. 9 ч. Второй сделает за это время 33 · 9 = 3/11 всей работы. Следовательно, на долю первого приходится ? ? 3/11 = 5/22. Его производительность 5/22 : 9 = 5/198 в час. Свою треть работы он выполнил за ? : 5/198 = 131/5 (ч), т. е. за 13 ч 12 мин.

Хотя решение выглядит намного красивее, чем первые два, его тоже трудно назвать существенно лучшим. Взгляните внимательно на уравнения второго решения, и вы заметите, что третье решение получено почти «дословным» пересказом этих уравнений.

Таким образом, на пути к решению задачи вас не должно останавливать большое число неизвестных, которые, по вашему мнению, следует ввести.

Однако старайтесь не вводить неизвестные, размерность которых не встречается в условии и не может быть получена как комбинация элементов условия. Введение таких неизвестных может усложнить задачу.

Вот простой пример.

Пример 2. Расстояние между двумя пунктами A и В пароход проходит по течению реки на а ч быстрее, чем то же расстояние в стоячей воде, и на b ч быстрее, чем против течения (b > а > 0). За какое время пароход проходит расстояние от A до В по течению?

Если ввести в рассмотрение неизвестные: v — скорость парохода в стоячей воде, w — скорость течения реки, x — расстояние, то получим систему двух уравнений с тремя неизвестными:

Найти из этой системы величину x/v + w можно, если сделать следующие преобразования:

и обозначить v/x = у, w/x = z. Мы придем к системе относительно у и z, решив которую, вычислим 1/y + z.

Однако такую систему можно было получить сразу, если бы мы не ввели в качестве неизвестного x пройденное пароходом расстояние.

В условии задачи не было чисел, выраженных в километрах, однако расстояние между пунктами являлось существенным связующим звеном. Это означает, что мы должны были принять его за единицу, а скорости v и w выражать в частях расстояния, пройденных за один час. В результате мы пришли бы к системе

которую не пришлось бы преобразовывать.

Разберем еще одну задачу, на примере которой видно, как решаются задачи на движение.

Пример 3. Из пункта С в пункт D выехал товарный поезд. Через 5 ч 5 мин навстречу ему из пункта D выехал пассажирский поезд. Они встретились в каком-то пункте А. После этого пассажирский поезд приехал в пункт С через 4 ч 6 мин, а товарный — в пункт D через 12 ч 55 мин. Сколько времени каждый поезд находился в пути?

Условия задачи можно отразить на схеме (рис. 18.1), где буквой В обозначено положение товарного поезда в момент выхода пассажирского из пункта D.

То обстоятельство, что оба поезда находились в точке А одновременно, мы отразим на схеме с помощью вертикального отрезка, связывающего оба пути. Схема подсказывает нам и выбор неизвестных. На путь от В до А товарный поезд потратил столько же времени, сколько пассажирский на путь от пункта D до А. Если обозначить это время через x, то на схеме не останется «белых пятен».

Пусть v1 — скорость товарного поезда, а v2 — скорость пассажирского поезда. Каждый из отрезков пути: от пункта С до А и от пункта D до А позволяет составить уравнения

1211/12v1 = хv2, (51/12 + x)v1 = 41/10v2.

Можно составить и уравнение для всего пути:

(51/12 + x + 1211/12)v1, = (x + 41/10) v2,

которое является следствием (точнее, суммой) первых двух уравнений. Однако это уравнение проще второго. Поэтому мы будем решать систему

Разделив первое уравнение на второе, получим

откуда x = 5 ч 10 мин (второй корень отрицательный и не имеет физического смысла). Итак, товарный поезд пройдет весь путь за 23 ч 10 мин, а пассажирский — за 9 ч 16 мин.

18.1. Бассейн наполняется четырьмя трубами за 4 ч. Первая, вторая и четвертая заполняют бассейн за 6 ч. Вторая, третья и четвертая — за 5 ч. За сколько времени заполняют бассейн первая и третья трубы?

18.2. У продавца испортились весы (плечи весов оказались неравными). Продавец отпустил покупателю два веса: первый раз на одну чашку весов положил килограммовую гирю, а на вторую — товар, во второй раз поменял гирю и товар местами. Компенсировал ли продавец неточность весов?

18.3. Школьник переклеивает свои марки в новый альбом. Если он наклеит по 20 марок на один лист, то ему не хватит альбома, если по 23 марки, то по крайней мере один лист останется пустым. Если школьнику подарить еще один такой же альбом, на каждом листе которого наклеено по 21 марке, то всего у него станет 500 марок. Сколько листов в альбоме?

18.4. Одному буксиру нужно перевезти за наименьшее время два понтона вниз по реке на l км. Было решено, что один понтон будет отправлен по течению реки самостоятельно, а другой будет некоторое время транспортироваться буксиром, после чего буксир оставит его и вернется за первым и отбуксирует его до конечного пункта. Сколько километров должен транспортироваться второй понтон, чтобы оба пришли к конечному пункту одновременно, и сколько потребуется времени на всю перевозку, если собственная скорость буксира v км/ч, а скорость течения реки u км/ч?

18. 5. Некто родился в девятнадцатом веке (до 1900 года). В 1901 году сумма цифр числа, выражающего год его рождения, равнялась сумме цифр числа, выражающего количество прожитых лет. Определите, в каком году родился некто.

18.6. Цена бриллианта пропорциональна квадрату его массы. Бриллиант массой p карат был разбит на две части, после чего его стоимость уменьшилась в k раз. Найдите массы частей, на которые был разбит бриллиант[12].

18.7. Некоторую часть маршрута туристам предстоит совершить вверх по реке. В их распоряжении моторная лодка, способная развивать две скорости с разным расходом горючего. Если скорость течения реки окажется равной u км/ч, то при движении на любой из собственных скоростей будет затрачено одинаковое количество горючего. Если же скорость течения в k раз больше (k > 1), то при движении с собственной скоростью v1 горючее будет израсходовано полностью, а при движении с собственной скоростью v2 останется A кг горючего. Какое количество горючего будет затрачено на весь путь?

18.8. У продавца мороженого есть по нескольку десятков порций мороженого пяти сортов — за 7, 9, 11, 13 и 15 p. Общее число порций равно 180, а общая стоимость — 1840 p. Порций мороженого по 7 и 9 p. вместе столько же, сколько по 11, 13 и 15 p. вместе. Кроме того, известно, что порций по 9 p. вдвое больше, чем по 15 p., и больше, чем по 13 p. Определите число порций каждого сорта.

18.9. Плоты шли из пункта A до устья реки вниз по течению. У устья реки их взял на буксир пароход и через 11,5 суток после выхода плотов из пункта A доставил их по озеру в пункт B. Сколько времени пароход вел плоты от устья реки по озеру до B, если известно, что пароход совершает рейс (без буксировки) от A до B за 40 ч и от B до A за 48 ч, а скорость во время буксировки уменьшается вдвое.

18.10. Три пловца должны проплыть из A в B и обратно. Сначала стартует первый, через 5 с — второй, еще через 5 с — третий. Некоторую точку C, находящуюся между пунктами A и B, все пловцы миновали одновременно (до этого времени ни один из них в B не побывал). Третий пловец, доплыв до B и повернув назад, встречает второго в 9 м от B, а первого — в 15 м от B. Найдите скорость третьего пловца, если расстояние AB равно 55 м.

18.11. Сосуд, содержащий p%?й раствор кислоты, долили доверху q%?м раствором кислоты и после перемешивания отлили то же количество. Проделав эту операцию k раз, получили r%?й раствор. Какую часть объема сосуда занимал первоначальный раствор[13]?

18.12. Из пункта A в пункт B выехал автомобиль. Одновременно из B навстречу ему выехал мотоцикл. Через некоторое время они встретились. В момент их встречи из B в A выехал второй мотоцикл и в некоторый момент времени встретился с автомобилем. Расстояние между пунктами первой и второй встреч равно 2/9AB. Если бы скорость автомобиля была на 20 км/ч меньше, то автомобиль встретился бы с первым мотоциклом через 3 ч после их выезда и расстояние между пунктами встреч было бы равно 60 км. Найдите AB, если скорости обоих мотоциклов одинаковы.

18.13. Пассажир, опоздавший на поезд, сначала решил догнать его на такси, однако через некоторое время пересел на автобус, заплатив за билет А p., и прибыл на одну из станций одновременно с поездом. Между тем он обнаружил, что если бы продолжал ехать на такси, то догнал бы поезд на ? ч раньше, истратив при этом на В p. меньше. Какова скорость поезда, если скорость такси v1 км/ч, автобуса v2 км/ч (v1 > v2), а стоимость проезда 1 км на такси а p.?

18.14. Товарный поезд, шедший из А в В, прибыл в С одновременно с пассажирским, шедшим из В в А со скоростью в m раз большей, чем скорость товарного поезда. Оба состава простояли t ч в пункте С, затем продолжили путь, причем каждый увеличил скорость на 25%. Товарный поезд прибыл в В на t1 ч позже, а пассажирский в А на t2 ч позже, чем если бы они двигались без остановки с первоначальной скоростью. Насколько раньше товарный поезд вышел из А, чем пассажирский из В?

18.15. Расстояние между пунктами А и В равно s км. Из пункта А в пункт В вылетел вертолет, а через t ч в том же направлении вылетел самолет. Самолет догнал вертолет в d км от А, долетел до В и сразу повернул обратно. В d км от В самолет встретил вертолет и вернулся в А позднее, чем вертолет прибыл в В. Насколько раньше вертолет прибыл в В, чем самолет вернулся в А?

18.16. В озеро впадают две реки. Пароход выходит из порта M на первой реке и плывет вниз по течению, затем через озеро (на озере течение отсутствует) и по второй реке вверх по течению до порта N. Придя в N, пароход отправляется в обратный путь.

Известна собственная скорость парохода v и скорости течения рек: v1 и v2. На путь от M до N, равный по длине s, и на обратный путь пароход тратит одинаковое время t. Какое расстояние пароход проходит по озеру?

18.17. Из пункта А в пункт В в 8 ч утра выходит скорый поезд. В этот же момент из В в А выходят пассажирский и курьерский поезда, причем скорость курьерского в два раза больше скорости пассажирского. Скорый поезд прибывает в В в 13 ч 50 мин того же дня, а встречает курьерский поезд не ранее 10 ч 30 мин утра. Когда пассажирский поезд прибудет в пункт А, если между моментами встреч скорого поезда с курьерским и скорого поезда с пассажирским проходит не менее часа?

18. 18. Завод должен получить 1100 деталей. На базе имеются комплекты по 70, 40 и 25 деталей. Стоимость пересылки одного комплекта равна соответственно 20, 10 и 7 p. Какие комплекты и в каком количестве следует заводу заказать, чтобы расходы по пересылке были наименьшими? Переупаковка комплектов на базе не допускается.

Решение задач на составление уравнений

Есина Наталья Васильевна

учитель математики

МБОУ СОШ №12

ст. Кавказская

Кавказский район

Урок в 11 классе по теме:

Решение задач на составление уравнений.

Цели урока:

учить решать задачи составлением уравнений на смеси, растворы, сплавы, движения и работу

Ход урока.

1. Организационый момент. Сообщение темы и цели урока.

Класс разбивается на группы.

2.Изучение нового материала.

Для решения задач по данной теме следует ввести следующие понятия. Смесь состоит из чистого вещества и примеси. Чистое вещество в каждой задаче определяется отдельно, а все остальные вещества относят к примеси. Доля чистого вещества в смеси – это отношение количества чистого вещества в смеси к общему количеству смеси: у=m/M, где доля чистого вещества равна отношению процентного содержания чистого вещества в смеси к ста процентам .При решении задач следует помнить, что при соединении( разъединении) смесей с одним и тем же чистым веществом количества чистого вещества и общие количества смесей складываются (вычитаются).

3.Основные этапы решения задач.

А) В качестве неизвестных величин выбирают те, которые требуется найти.

Б) Из веществ в задаче, выбирается одно в качестве чистого вещества, при, если у- доля чистого вещества, то( 1-у)- доля примеси.

В) Если в задаче имеются процентные содержания, их следует перевести в доли.

Процент- сотая часть. Например, 4% равны 0,04; 60% равны 60:100 = 0,6 и т.д.

Г) Описывать изменение смеси с помощью таблиц с помощью 3 основных величин m, M,у.

Д ) Составить уравнение : m=у· M .

Е) Решение уравнения.

4.Примеры решения задач.

Задача 1. Сплавили 400г сплава меди и олова, содержащего 70% олова и 600 г сплава меди и олова, содержащего 80% олова. Сколько процентов олова в получившейся смеси?

Решение. Пусть у % олова в получившейся смеси.

Составим таблицу.

Сплав

m(г)

M(г)

у

1

400·0,7

400

0,7

2

600· 0,8

600

0,8

1+2

400· 0,7 +600· 0,8

1000

0,01у

Составим уравнение используя 3 строку и решим его.

400· 0,7+ 600· 0,8 = 1000· 0,01у;

280 + 480= 10у;

760 =10у;

У = 76

Ответ: 76 %

Задача 2. Имеются 2, в первом из которых содержится 30% ,а во втором 50%серебра.Сколько килограммов первого сплава необходимо добавить к 40 кг второго сплава , чтобы получить сплав , содержащий 40 % серебра?

Решение. Пусть взяли х кг первого сплава.

Составим таблицу.

сплав

m(кг)

M(кг)

у

1

0,3х

х

0,3

2

0,2 ·40

40

0,5

1+2

0,3+0,2·40

Х+40

0,4

Составим уравнение по 3 строке таблицы.

0.3х+ 0.5· 40 = (х+40)·0,4;

0,3х+20= 0,4х +16;

-0,1 х =-4;

х = 40

Ответ : 40 кг.

Задача 3. Имеются два раствора цемента, состоящих из воды , песка и цемента.

Известно, что первый раствор содержит 10 % воды, а второй 40% цемента Процентное содержание песка в первом растворе в два раза больше, чем во втором.

Смешав 300 кг первого раствора и 400 кг второго раствор, получили новый раствор, в котором оказалось 30% песка. Сколько килограммов цемента содержится в получившемся растворе?

смесь

m(кг)

M(кг)

у

1

Вода 300·0,1

Цемент 300(1-(0,1+2х)

Песок 2х·300

300

0,1

1-(0,1+2х)

2

Вода 1-(0,4+х)·400

Цемент 0,4·400

песок 400х

400

1-(0,4+х)

0,4

х

1+2

Вода 30+(0,6-х)·400

Цемент300(0,9- 2х)+160

Песок600х+400х

700

?

?

0,3

Решение. Составим и решим уравнение 600х+400х =0,3·700;

Х= 0,21

Доля песка в первом сплаве 0,42 , а во втором сплаве 0,21.Теперь в качестве чистого вещества выберем цемент и пусть у-доля цемента в получившемся сплаве .Посчитаем долю цемента в первом сплаве: 1- 0,52= 0,48

смесь

mкг)

M(кг)

у

1

0.48· 300

300

0,48

2

0.4·400

400

0,4

1+2

0. 48·300+ 0.4· 400

700

у

Составим и вычислим выражение по последней строке 0,48· 300+ 0,4· 400 = 144+160 = 304

Ответ: 304 кг.

5. Задачи для самостоятельной работы

Задача 1. Сплавили 4 кг сплава цинка и меди, содержащего 40 % цинка и 6 кг сплава

цинка и меди, содержащего 20 % цинка. Найти процентную концентрацию меди в получившемся сплаве.

Задача 2. В смеси ацетона и спирта ацетона в 2 раза меньше , чем спирта. Когда к этой смеси добавили 300л спирта , получили смесь с процентным содержанием ацетона 28 % Сколько литров ацетона было в смеси первоначально?

Задача 3 .В первой канистре находится 5% раствор соли , а во второй канистре- 10% В пустое ведро выливают половину раствора из каждой канистры в результате ведро содержит7 % раствор Во сколько раз масса раствора в первой канистре больше массы раствора во второй?

Задача 4. Имеется кусок сплава меди с оловом общей массой 24 кг , содержащий 45% меди. Сколько чистого олова надо прибавить к этому куску сплава, чтобы полученный раствор содержал 40% меди?

Задача5.У ювелира два одинаковых по массе слитка , в одном из которых 36% золота, а

в другом 64%. Сколько процентов золота содержится в сплаве, полученном из этих слитков?

6. Проверяется решение. Подводятся итоги.

7.Дома :Карточки и тесты

Решение проблем | Начальная алгебра

Цели обучения

  • Определить процесс решения задач
    • Перевод слов в алгебраические выражения и уравнения
    • Определить процесс решения текстовых задач
  • Решить задачи, содержащие ставки
    • Применение шагов решения текстовых задач к задачам на расстояние, скорость и время
    • Применить шаги для решения текстовых задач к задачам с процентными ставками
    • Вычислить формулу, используя замену
    • Переупорядочить формулы, чтобы изолировать определенные переменные
    • Определить неизвестное по формуле
  • Решение дополнительных приложений линейных уравнений
    • Применение шагов решения текстовых задач к задачам по геометрии
    • Используйте формулу для перевода градусов Фаренгейта в градусы Цельсия
    • Вычислить формулу с помощью подстановки
    • Переупорядочить формулы, чтобы изолировать определенные переменные
    • Определить неизвестное по формуле

Определите процесс решения проблем

Проблемы с Word могут быть сложными. Часто требуется немного практики, чтобы преобразовать английское предложение в математическое предложение, что является одним из первых шагов к решению текстовых задач. В приведенной ниже таблице слова или фразы, обычно связанные с математическими операторами, распределены по категориям. Текстовые задачи часто содержат эти или похожие слова, поэтому полезно посмотреть, какие математические операторы с ними связаны.

Дополнение [латекс]+[/латекс] Вычитание [латекс]-[/латекс] Умножение [латекс]\раз[/латекс] Переменная ? равно [латекс]=[/латекс]
Более Менее Двойной Номер Является
Вместе В прошлом Продукт Часто значение, для которого не предоставляется никакой информации. То же, что и
Сумма медленнее, чем умножить на Через сколько часов?
Итого остаток от Сколько это будет стоить?
В будущем  разница
быстрее, чем

Вот несколько примеров:

  • [латекс]х\текст{ равно }5[/латекс] становится [латекс]х=5[/латекс]
  • На три больше числа становится [латекс]х+3[/латекс]
  • Число на четыре меньше числа становится [латекс]х-4[/латекс]
  • Двойная стоимость становится [latex]2\cdot\text{ cost }[/latex]
  • Продукты и бензин вместе на неделю стоят 250 долларов, что означает [латекс]\текст{ продукты }+\текст{ газ }=250[/латекс]
  • Разница между 9 и числом становится [латекс]9-х[/латекс]. Обратите внимание, что 9 стоит первым в предложении, а выражение

Давайте потренируемся переводить еще несколько английских фраз в алгебраические выражения.

Пример

Переведите таблицу в алгебраические выражения:

 некоторое число  сумма числа и 3  удвоенная сумма числа и 3
длина  удвоить длину  удвоить длину, уменьшить на 6
стоимость  разница стоимости и 20  2-кратная разница стоимости и 20
 некоторое количество  разность 5 и количество   разность 5 и количества, деленная на 2
количество времени  втрое больше времени  втрое больше времени, увеличено на 5
 на расстоянии  сумма [латекс]-4[/латекс] и расстояние  сумма [латекс]-4[/латекс] и удвоенного расстояния

Показать решение

В этом видео-примере мы покажем, как перевести больше слов в математические выражения.

 

Сила алгебры заключается в том, что она может помочь вам смоделировать реальные ситуации, чтобы ответить на вопросы о них.

Вот несколько шагов, чтобы перевести проблемные ситуации в алгебраические уравнения, которые вы можете решить. Задача на каждые слов не идеально вписывается в эти шаги, но они помогут вам начать работу.

  1. Прочтите и поймите проблему.
  2. Определите константы и переменные в задаче.
  3. Перевод слов в алгебраические выражения и уравнения.
  4. Напишите уравнение, представляющее проблему.
  5. Решите уравнение.
  6. Проверьте и интерпретируйте свой ответ. Иногда помогает написание предложения.

Пример

Двадцать восемь меньше, чем в пять раз определенное число равно 232. Что это за число?

Показать решение

В следующем видео мы покажем еще один пример того, как перевести предложение в математическое выражение с помощью метода решения задач.

Другой тип числовой задачи связан с последовательными числами. Последовательные числа — это числа, которые идут одно за другим, например 3, 4, 5. Если мы ищем несколько последовательных чисел, важно сначала определить, как они выглядят с переменными, прежде чем мы начнем уравнение.

Например, предположим, что я хочу узнать следующее целое число после 4. С точки зрения математики, мы должны добавить 1 к 4, чтобы получить 5. Мы можем обобщить эту идею следующим образом: целое число, следующее за любым числом, x , это [латекс]х+1[/латекс]. Если мы продолжим этот шаблон, мы сможем определить любое количество последовательных целых чисел из любой начальной точки. В следующей таблице показано, как описать четыре последовательных целых числа с помощью алгебраической записи.

Первый [латекс]х[/латекс]
Второй [латекс]x+1[/латекс]
Третий [латекс]x+2[/латекс]
Четвертый  [латекс]x+3[/латекс]

Мы применяем идею последовательных целых чисел для решения текстовой задачи в следующем примере.

Пример

Сумма трех последовательных целых чисел равна 93. Что это за числа?

Показать решение

Ставки

Часто существует хорошо известная формула или соотношение, применимое к словесной задаче. Например, если вы планируете поездку на автомобиле, вам нужно знать, сколько времени вам потребуется, чтобы добраться до пункта назначения. [latex]d=rt[/latex] — это хорошо известное соотношение, которое связывает пройденное расстояние, скорость, с которой вы путешествуете, и продолжительность путешествия.

Расстояние, скорость и время

Если известны две величины в соотношении [latex]d=rt[/latex], третью можно легко найти, используя методы решения линейных уравнений. Например, если вы знаете, что будете ехать по дороге с ограничением скорости [латекс]30\frac{\text{миль}}{\текст{час}}[/латекс] в течение 2 часов, вы можете найти расстояние, которое вы проедете, умножив скорость на время или [латекс]\влево(30\frac{\text{миль}}{\text{час}}\вправо)\влево(2\текст{часы}\вправо)= 60\text{ миль }[/latex].

Мы можем обобщить эту идею в зависимости от того, какую информацию нам дают и что мы ищем. Например, если нам нужно найти время, мы можем решить уравнение [latex]d=rt[/latex] для t , используя деление:

[latex]d=rt\\\frac{d}{r} =t[/latex]

Аналогично, если мы хотим найти скорость, мы можем выделить r с помощью деления:

[latex]d=rt\\\frac{d}{t}=r[/latex]

В следующих примерах вы увидите, как эта формула применяется для ответа на вопросы о сверхмарафонском беге.

Энн Трейсон

Ультрамарафонский бег (определяемый как любой забег длиннее 26,2 миль) становится очень популярным среди женщин, хотя остается нишевым видом спорта, в котором доминируют мужчины. Энн Трейсон за свою карьеру побила двадцать мировых рекордов. Одним из таких рекордов стал забег на выносливость на 50 миль по реке Американ-Ривер, который начинается в Сакраменто, штат Калифорния, и заканчивается в Оберне, штат Калифорния. [1] В 1993 году Трэйсон финишировал со временем 6:09:08. Рекорд среди мужчин на той же трассе был установлен в 1994 Тома Джонсона, завершившего дистанцию ​​со временем 5:33:21. [2]

В следующих примерах мы будем использовать формулу [latex]d=rt[/latex], чтобы ответить на следующие вопросы о двух полозьях.

  1. Каков был показатель рекордных забегов каждого бегуна?
  2. К тому времени, когда Джонсон закончил, сколько еще миль должен был пробежать Трейсон?
  3. Насколько дальше мог бы бежать Джонсон, если бы он бежал так же долго, как Трейсон?
  4. Сколько времени каждый бегун пробежал одну милю?

Чтобы упростить ответы на вопросы, мы округлим время двух бегунов до 6 часов и 5,5 часов.

Пример

Каков был показатель рекордных забегов каждого бегуна?

Показать решение

Теперь, когда мы знаем скорость каждого бегуна, мы можем ответить на второй вопрос.

Пример

К тому времени, когда Джонсон финишировал, сколько еще миль должен был пробежать Трейсон?

Показать решение

Третий вопрос аналогичен второму. Теперь, когда мы знаем скорость каждого бегуна, мы можем ответить на вопросы об отдельных дистанциях или времени.

Примеры

Насколько дальше мог бы пробежать Джонсон, если бы он бежал так же долго, как Трейсон?

Показать решение

Теперь мы займемся последним вопросом, где нас просят найти время для каждого бегуна.

Пример

За какое время каждый бегун пробежал одну милю?

Показать решение

В следующем видео мы показываем еще один пример ответа на многие вопросы о скорости с учетом расстояния и времени.

Простые проценты

Чтобы побудить клиентов вкладывать свои деньги, многие банки предлагают процентные счета. Счета работают следующим образом: клиент вносит определенную сумму денег (называемую основной суммой, или P ), которая затем медленно растет в соответствии с процентной ставкой ( R , измеряется в процентах) и продолжительностью времени (). T , обычно измеряется в месяцах), что деньги остаются на счете. Сумма, заработанная с течением времени, называется процентами ( I ), которые затем передаются покупателю.

Осторожно! Процентные ставки обычно указываются как годовые, но также могут быть ежемесячными, ежеквартальными, раз в два месяца или даже в течение определенного периода времени. Важно, чтобы единицы времени и единицы процентной ставки совпадали. Вы увидите, почему это важно, в следующем примере.

Самый простой способ рассчитать проценты по счету — использовать формулу [latex]\displaystyle I=P\,\cdot \,R\,\cdot \,T[/latex].

Если мы знаем любую из трех величин, связанных с этим уравнением, мы можем найти четвертую. Например, если мы хотим найти время, необходимое для начисления определенной суммы процентов, мы можем найти T, используя деление:

[латекс]\displaystyle\begin{array}{l}\,\,\,\,\,\,\,\,\,\,\,\,I=P\,\cdot \,R\ ,\cdot \,T\\\\ \frac{I}{{P}\,\cdot \,R}=\frac{P\cdot\,R\,\cdot \,T}{\,P\ ,\cdot \,R}\\\\\,\,\,\,\,\,\,\,\,\,\,{T}=\frac{I}{\,R\,\cdot \,T}\end{array}[/latex]

Ниже приведена таблица, показывающая результат решения для каждой отдельной переменной в формуле.

Решить для Результат
Я [латекс]I=P\,\cdot \,R\,\cdot \,T[/латекс]
Р [латекс]{P}=\frac{I}{{R}\,\cdot \,T}[/латекс]
Р [латекс]{R}=\frac{I}{{P}\,\cdot \,T}[/латекс]
Т  [латекс]{T}=\frac{I}{{P}\,\cdot \,R}[/латекс]

В следующих примерах мы покажем, как подставить заданные значения в формулу простых процентов и расшифровать, для какой переменной нужно найти.

В следующем видео показан еще один пример определения остатка на счете через заданное время, вложенную основную сумму и ставку.

В следующем примере вы увидите, почему важно убедиться, что единицы процентной ставки соответствуют единицам времени при использовании формулы простых процентов.

Дальнейшие применения линейных уравнений

Формулы встречаются во многих сферах жизни. Мы видели формулу, которая связывает расстояние, скорость и время, и формулу простого процента на инвестиции. В этом разделе мы подробнее рассмотрим формулы и увидим примеры формул для размеров геометрических фигур, а также формулу для преобразования температуры между градусами Фаренгейта и градусами Цельсия.

Геометрия

Существует много геометрических фигур, которые хорошо изучены на протяжении многих лет. Мы довольно много знаем о кругах, прямоугольниках и треугольниках. Математики доказали множество формул, описывающих размеры геометрических фигур, включая площадь, периметр, площадь поверхности и объем.

Периметр

Периметр — это расстояние вокруг объекта. Например, рассмотрим прямоугольник с длиной 8 и шириной 3. В прямоугольнике две длины и две ширины (противоположные стороны), поэтому мы добавляем [латекс]8+8+3+3=22[/латекс ]. Поскольку в прямоугольнике две длины и две ширины, периметр прямоугольника можно найти по формуле [латекс]{P}=2\left({L}\right)+2\left({W}\right )[/латекс], где

L = длина

W = ширина

В следующем примере мы будем использовать разработанный нами метод решения задач для нахождения неизвестной ширины с помощью формулы для периметра прямоугольника. Подставив известные нам размеры в формулу, мы сможем выделить неизвестную ширину и найти наше решение.

В этом видео показана аналогичная проблема с садовым ящиком.

Мы могли бы выделить w в формуле для периметра, прежде чем решать уравнение, и если бы мы собирались использовать формулу много раз, это могло бы сэкономить много времени. В следующем примере показано, как изолировать переменную в формуле перед заменой известных размеров или значений в формулу.

Пример

Выделить член, содержащий переменную w, , из формулы для периметра прямоугольника :  

[латекс]{P}=2\left({L}\right)+2\ влево({W}\вправо)[/латекс].

Показать решение

Площадь

Площадь треугольника определяется как [латекс] A=\frac{1}{2}bh[/латекс], где

A = площадь
b = длина основания
h = высота треугольника

Помните, что когда две переменные или число и переменная расположены рядом друг с другом без математического оператора между ними, вы можете предположить, что они перемножаются. Это может показаться разочаровывающим, но вы можете думать об этом как о математическом сленге. За прошедшие годы люди, часто использующие математику, просто сделали это сокращение настолько, что оно стало общепринятым.

В следующем примере мы будем использовать формулу площади треугольника, чтобы найти недостающее измерение, а также использовать подстановку, чтобы найти основание треугольника, зная площадь и высоту. {2}[/латекс], через высоту, ч .

В этом примере переменная h довольно глубоко запрятана в формуле площади поверхности цилиндра. Используя порядок операций, его можно выделить. Прежде чем вы посмотрите на решение, используйте поле ниже, чтобы записать то, что вы считаете лучшим первым шагом, чтобы изолировать h .

Показать решение


  1. «Энн Трейсон». Википедия. По состоянию на 5 мая 2016 г. https://en.wikipedia.org/wiki/Ann_Trason. ↵
  2.  «Забег на выносливость на 50 миль по реке Американ-Ривер». Википедия. По состоянию на 5 мая 2016 г. https://en.wikipedia.org/wiki/American_River_50_Mile_Endurance_Run. ↵

Решение двухшаговых уравнений — ChiliMath

Поиск

Нет никаких сомнений в том, что решить двухшаговое уравнение чрезвычайно просто. Как следует из названия, двухшаговые уравнения можно решить всего за два шага. Если вы впервые сталкиваетесь с двухшаговыми уравнениями, не волнуйтесь, потому что мы рассмотрим достаточно примеров, чтобы вы познакомились с процессом.

При решении уравнения в целом мы всегда помним о том, что все, что мы делаем с одной частью уравнения, должно быть сделано и с другой, чтобы уравнение оставалось сбалансированным.

Мы знаем, что полностью решили двухшаговое уравнение, если переменная, обычно представленная буквой в алфавите, изолирована на одной стороне уравнения (слева или справа), а число расположено на противоположной стороне .


ОБЫЧНЫЙ способ решения двухшагового уравнения:

Примечание : Это «обычный» метод, потому что таким образом решается большинство двухшаговых уравнений. Обратите внимание, что шаг 2 можно заменить на шаг 3, который по сути такой же.

1)  Сначала добавьте или вычтите обе части линейного уравнения на одно и то же число.

2)  Во-вторых, умножьте или разделите обе части линейного уравнения на одно и то же число.

3)* Вместо шага №2 всегда умножайте обе части уравнения на обратную величину коэффициента переменной.


Пример 1: Решите приведенное ниже двухэтапное уравнение.

Как следует из названия этого линейного уравнения, для решения неизвестной переменной требуется два шага. Как правило, первый шаг заключается в избавлении от числа, «самого дальнего» от члена с решаемой переменной. Затем мы исключаем число, «ближайшее» к переменной. Число либо умножает, либо делит переменную. Его также называют коэффициентом срока.

Переменная здесь x. Наша цель состоит в том, чтобы решить x, изолируя его на одной стороне уравнения. Сохранение переменной слева или справа не имеет никакого значения. Это тебе решать! В этой задаче давайте оставим его слева, так как он уже там.

В той части (левая часть линейного уравнения), где находится переменная, обратите внимание, что 2 «ближайшая» к переменной x, а 5 «самая дальняя».

Это простое наблюдение позволяет нам решить, какое число исключить первым. Это, очевидно, +5, потому что это дальше между ними. Противоположность +5 равна -5, это означает, что мы вычтем обе части уравнения на 5.

После исключения 5 в левой части уравнения путем вычитания обеих частей на 5, пришло время избавиться от ближайшего числа или непосредственно присоединен к x, который равен 2 в 2x. Поскольку 2 — это умножение переменной x, противоположная ей операция — деление на 2.

Разделив обе части на 2, мы получим окончательный ответ или решение данного двухшагового линейного уравнения.

Напомню, задача считается решенной, потому что коэффициент переменной просто положительный, +1.


Пример 2: Решите приведенное ниже двухэтапное уравнение.

Наша цель — сохранить переменную x в одной части уравнения. Неважно, с какой стороны, однако «стандартной» практикой является сохранение решаемой переменной слева. Некоторые учителя алгебры могут потребовать, чтобы переменная оставалась слева, и в этом нет ничего страшного. Лично я не возражаю, где вы держите переменную, слева или справа, если изолированная переменная в одной части уравнения имеет коэффициент +1.

На первом этапе из переменной x удаляется «самое дальнее» число. Обратите внимание, что -3 «ближе всего» к x, а -8 «дальше». Таким образом, мы можем исключить -8, прибавив к его противоположности +8.

На втором этапе нужно избавиться от числа, ближайшего к переменной x, которая равна -3. Поскольку -3 умножает переменную x, противоположная ей операция — деление на -3. Разделив обе части на -3, мы решили линейное уравнение.

Быстрое напоминание: -3 разделить на -3 равно +1.


Пример 3: Решите приведенное ниже двухэтапное уравнение.

Вот ситуация, когда мы можем изолировать переменную x в правой части уравнения, поскольку она уже там.

Глядя на правую часть уравнения, где расположена переменная, число 3 ближе всего к x, потому что 3 делится на переменную x. С другой стороны, число 26 находится «дальше». Это означает, что нам придется иметь дело с +26, вычитая обе части уравнения на 26. Причина, по которой мы вычитаем, заключается в том, что аддитивное значение, обратное +26, равно -26.

Второй шаг — избавиться от знаменателя 3. Поскольку 3 — это деление x, его противоположная операция — умножение на 3.

После умножения обеих частей на 3 мы получили окончательный ответ. Вы можете переписать свой окончательный ответ как x = -9.


Пример 4: Решите приведенное ниже уравнение из двух.

Может показаться, что уравнение состоит из нескольких шагов, но это не так. Ее можно решить в два этапа. Не беспокойтесь о дробях, потому что с ними очень легко иметь дело. В этом случае вы будете применять правило сложения дробей. Правило гласит, что если вы складываете две дроби с одинаковым знаменателем, просто добавьте числители, а затем скопируйте общий знаменатель.

Вернемся к решению двухшагового уравнения выше, чтобы удалить дробь в левой части, которая равна \Large{ — {3 \over {10}}}, мы добавим \Large{{3 \over {10}} } к обеим частям уравнения.

Причина, по которой мы складываем, а не вычитаем, заключается в том, что аддитивная обратная величина \Large{ — {3 \over {10}}} равна \Large{+ {3 \over {10}}} .

После добавления \Large{{3 \over {10}}} к обеим сторонам слева останется только {\Large{{2 \over 5}}}x.

Для правой части уравнения имеем \Large{{9\более {10}} + {3 \более {10}} = {{12} \более {10}}}.

Все, что я сказал выше, это только первый шаг. Теперь переходим ко второму шагу. Посмотрите на коэффициент переменной х. Это \Large{{2 \over 5}}, что означает, что его обратная величина равна \Large{{5 \over 2}}.

Чтобы окончательно решить данное уравнение, мы умножим обе части уравнения на обратную величину коэффициента рассматриваемой переменной. Вот полное пошаговое решение:


Вам также может быть интересно:

Упражнения с двухшаговыми уравнениями с ответами

Упражнения с многошаговыми уравнениями с ответами

Решение линейных уравнений с одной переменной

Линейные уравнения с одной переменной — это уравнения, в которых переменная имеет показатель степени 1, что обычно не отображается (подразумевается). Примером может быть что-то вроде \(12x = x — 5\). Для решения линейных уравнений есть одна основная цель: выделить переменную . В этом уроке мы рассмотрим, как это делается на нескольких примерах.

Содержание

  1. Примеры решения одношаговых уравнений
  2. Примеры решения двухшаговых уравнений
  3. Примеры уравнений, которые сначала нужно упростить
  4. Бесконечно много или нет решений
  5. Резюме

реклама

Примеры решения одношаговых линейных уравнений

После всей вашей тяжелой работы по решению уравнения вы знаете, что вам нужен окончательный ответ, например \(x=5\) или \(y=1\). В обоих этих случаях переменная равна изолированный или сам по себе.

Итак, нам нужно выяснить, как изолировать переменную. Как мы это делаем, зависит от самого уравнения! Если умножили на что-то, будем делить. Если к нему что-то прибавилось, то мы его вычтем. Делая это, мы постепенно будем получать переменную саму по себе.

Давайте рассмотрим пример, чтобы увидеть, как это работает.

Пример

Решите уравнение: \(4x = 8\)

Решение

В этом примере 4 умножается на \(x\). Следовательно, чтобы изолировать \(x\), вы должны разделить эту сторону на 4. При этом вы должны помнить одно важное правило: все, что вы делаете с одной частью уравнения, вы должны делать и с другой стороной. Итак, мы разделим обе части на 4.

\(\begin{align}4x &= 8 \\ \dfrac{4x}{\color{red}{4}} &= \dfrac{8}{\color{red}{4}}\end{align} \)

Упрощение:

\(х = \в коробке{2}\)

Вот и все, один шаг и готово. (Вот почему такие уравнения часто называют «одношаговыми» уравнениями)

Проверка

Каждый раз, когда вы решаете линейные уравнения, вы всегда можете проверить свой ответ, подставив его обратно в уравнение. Если вы получаете верное утверждение, то ответ правильный. Это не обязательно для каждой задачи на 100%, но это хорошая привычка, поэтому мы сделаем это для наших уравнений.

В этом примере исходное уравнение было \(4x = 8\). Чтобы проверить это, убедитесь, что верно следующее:

\(\begin{align}4x &= 8\\ 4(2) &= 8 \\ 8 &= 8\end{align}\)

Это верное утверждение, поэтому наш ответ правильный.

В любом уравнении любая операция, которую вы выполняете с одной стороны, должна быть проделана и с другой стороной.

Давайте попробуем еще пару примеров, прежде чем переходить к более сложным уравнениям.

Пример

Решите: \(3x=12\)

Решение

Так как \(x\) умножается на 3, план состоит в том, чтобы разделить на 3 с обеих сторон:

\(\begin{align}3x &=12\\ \dfrac{3x}{\color{red}{3}} &=\dfrac{12}{\color{red}{3}}\\ x&= \ в штучной упаковке{4}\end{align}\)

Проверить

Чтобы проверить наш ответ, мы допустим \(x = 4\) и подставим его обратно в уравнение:

\(\begin{align}3x &= 12\\3(4) &= 12 \\ 12 &= 12\end{align}\)

Как и раньше, поскольку это верное утверждение, мы знаем, что наш ответ правильный.

В следующем примере вместо умножения переменной на значение из переменной вычитается значение. Чтобы «отменить» это, мы добавим это значение к обеим сторонам.

Пример

Решите: \(y-9=21\)

Решение

На этот раз из y вычитается 9. Итак, мы отменим это, добавив 9 к обеим сторонам.

\(\begin{align}y-9&=21\\ y-9 \color{red}{+9}&=21\color{red}{+9}\\y&=30\end{align}\)

Далее мы рассмотрим так называемые «двухшаговые» уравнения. В этих уравнениях нам нужно будет отменить две операции, чтобы изолировать переменную.

Примеры двухшаговых уравнений

В каждом из приведенных выше примеров нужно было выполнить один шаг, прежде чем мы получили ответ. В следующих примерах вы увидите, как работать с уравнениями, состоящими из двух шагов. Если есть более одной операции, важно помнить порядок операций, PEMDAS. Поскольку вы отменяете операции над \(x\), вы будете работать «снаружи внутрь». Это легче понять, когда вы видите это на примере.

Пример

Решите: \(2x-7=13\)

Решение

Обратите внимание на две операции, происходящие с \(x\): оно умножается на 2, а затем из него вычитается 7. Нам нужно будет отменить их. Но только \(x\) умножается на 2, поэтому первым шагом будет добавление 7 к обеим сторонам. Тогда мы можем разделить обе стороны на 2.

Прибавляя 7 к обеим сторонам:

\(\begin{align} 2x-7 &= 13\\ 2x-7 \color{red}{+7} & =13 \color{red}{+7}\\ 2x&=20\end{align}\ )

Теперь разделите обе части на 2:

\(\begin{align} 2x &=20 \\ \dfrac{2x}{\color{red}{2}}&=\dfrac{20}{\color{red}{2}}\\ x&= \ в коробках{10}\end{align}\)

Проверка

Как и в случае с более простыми задачами, вы можете проверить свой ответ, подставив свое значение \(x\) обратно в исходное уравнение.

\(\begin{align}2x-7&=13\\ 2(10) – 7 &= 13\\ 13 &= 13\end{align}\)

Это правда, значит, у нас есть правильный ответ.

Давайте рассмотрим еще один двухшаговый пример, прежде чем снова перейти к сложности. Убедитесь, что вы понимаете каждый показанный шаг, а также проработайте проблему.

Пример

Решите: \(5w + 2 = 9\)

Решение

Как и выше, есть две операции: \(w\) умножается на 5, а затем к нему прибавляется 2. Мы отменим их, сначала вычитая 2 с обеих сторон, а затем разделив на 5.

\(\begin{align}5w + 2 &= 9\\ 5w + 2 \color{red}{-2} &= 9 \color{red}{-2}\\ 5w &= 7\\ \dfrac{ 5w}{\color{red}{5}} &=\dfrac{7}{\color{red}{5}}\\w=\boxed{\dfrac{7}{5}}\end{align} \)

Дробь справа не может быть упрощена, так что это наш окончательный ответ.

Проверить

Пусть \(w = \dfrac{7}{5}\). Тогда:

\(\begin{align}5w + 2 &= 9\\ 5\left(\dfrac{7}{5}\right) + 2 &= 9\\ 7 + 2 &= 9\\ 9 &= 9 \ end{align}\)

Итак, у нас снова правильный ответ!

Упрощение перед решением

В следующих примерах есть больше переменных терминов и, возможно, необходимо некоторое упрощение. В каждом случае шаги будут заключаться в том, чтобы сначала упростить обе стороны, а затем использовать то, что мы делали, чтобы изолировать переменную. Сначала мы подробно рассмотрим пример, чтобы увидеть, как все это работает.

Чтобы понять этот раздел, вам должно быть удобно сочетать подобные термины.

Пример

Решите: \(3x+2=4x-1\)

Решение

Поскольку обе части упрощены (нет скобок, которые нужно вычислять, и одинаковых терминов, которые нужно комбинировать), следующим шагом будет получение всех x на одной стороне уравнения и всех чисел на другой. другая сторона. Применяется то же правило: что бы вы ни делали с одной частью уравнения, вы должны делать и с другой стороной!

Можно переместить \(3x\) или \(4x\). Предположим, вы переместили \(4x\). Поскольку оно положительное, вы бы сделали это, вычитая его из обеих сторон:

. \(\begin{align}3x+2 &=4x-1\\ 3x+2\color{red}{-4x} &=4x-1\color{red}{-4x}\\ -x+2 & =-1\конец{выравнивание}\)

Теперь уравнение выглядит как те, что были обработаны ранее. Следующий шаг — вычесть 2 с обеих сторон:

. \(\begin{align}-x+2\color{red}{-2} &= -1\color{red}{-2}\\-x=-3\end{align}\)

Наконец, поскольку \(-x= -1x\) (это всегда верно), разделите обе части на \(-1\):

\(\begin{align}\dfrac{-x}{\color{red}{-1}} &=\dfrac{-3}{\color{red}{-1}}\\ x&=3\end {выровнять}\)

Проверить

Вы должны воспользоваться моментом и убедиться, что следующее утверждение верно:

\(3(3)+ 2 = 4(3) – 1\)

В следующем примере нам нужно будет использовать распределительное свойство перед решением. Здесь легко ошибиться, поэтому убедитесь, что вы распределили число перед скобками на все термины внутри.

Пример

Решите: \(3(х+2)-1=х-3(х+1)\)

Решение

Сначала распределите числа 3 и –3 и соберите одинаковые члены.

\(\begin{align} 3(x+2)-1 &=x-3(x+1)\\ 3x+6-1&=x-3x-3 \\ 3x+5&=-2x-3\end {выровнять}\)

Теперь мы можем добавить 2x к обеим сторонам. (Помните, что вы получите тот же ответ, если вместо этого вычтете 3x с обеих сторон)

\(\begin{align} 3x+5\color{red}{+2x} &=-2x-3\color{red}{+2x}\\ 5x+5& =-3\end{align}\)

Отсюда мы можем решить, как и с другими двухшаговыми уравнениями.

\(\begin{align}5x+5\color{red}{-5} &=-3\color{red}{-5}\\ 5x &=-8\\ \dfrac{5x}{\color{ red}{5}}&=\dfrac{-8}{\color{red}{5}}\\ x &= \dfrac{-8}{5} \\ &=\boxed{-\dfrac{8 }{5}}\end{align}\)

Проверить

Это был трудный вопрос, поэтому не забудьте проверить свой ответ и убедиться, что вы не ошиблись. Для этого вам необходимо убедиться, что следующее утверждение верно:

. \(3\влево(-\dfrac{8}{5}+2\вправо)-1=\влево(-\dfrac{8}{5}\вправо)-3\влево(-\dfrac{8}{ 5}+1\вправо)\)

(Примечание: это действительно работает, но вы должны быть очень осторожны со скобками!)

Бесконечное множество решений и ни одного решения

Бывают случаи, когда вы выполняете все эти шаги, и приходит действительно странное решение. Например, при решении уравнения \(x+2=x+2\), используя описанные выше шаги, в итоге получится \(0=0\). Это, конечно, верно, но что хорошего в этом?

Если вы получаете подобное утверждение, это означает, что уравнение имеет бесконечно много решений. Любой \(x\), который вы можете придумать, будет удовлетворять уравнению \(x+2=x+2\). Правильный ответ в этом случае — «бесконечно много решений».

Другая ситуация возникает, когда вы упрощаете уравнение до утверждения, которое никогда не бывает истинным, например, \(3=4\) или \(0=1\). Это происходит с уравнением \(x+5=x-7\), которое приводит к \(5=-7\), что, безусловно, никогда не бывает верным. Это означает, что никакой \(x\) не удовлетворял бы этому уравнению. Другими словами, «нет решения». Итого:

  • Если вы получите утверждение, которое всегда истинно, например \(5 = 5\) или \(0 = 0\), то существует бесконечно много решений.
  • Если вы получаете утверждение, которое всегда ложно, например \(10 = 11\) или \(1 = 5\), то решений нет.

реклама

Резюме

Решение линейных уравнений заключается в выделении переменной. В зависимости от уравнения это может занять всего один шаг или много шагов. Всегда проверяйте, нужно ли вам сначала упростить одну или обе части уравнения, и всегда проверяйте свой ответ.

Подпишитесь на нашу рассылку!

Мы всегда публикуем новые бесплатные уроки и добавляем новые учебные пособия, руководства по калькуляторам и наборы задач.

Подпишитесь, чтобы время от времени получать электронные письма (раз в пару или три недели), сообщающие вам о новинках!

2.4 Использование общей стратегии решения линейных уравнений — Элементарная алгебра 2e

Цели обучения

К концу этого раздела вы сможете:

  • Решение уравнений с использованием общей стратегии
  • Классифицировать уравнения

Приготовься 2,8

Прежде чем приступить к работе, пройдите этот тест на готовность.

Упрощение: −(a−4).−(a−4).
Если вы пропустили эту проблему, просмотрите пример 1.137.

Приготовься 2,9

Умножить: 32(12x+20)32(12x+20).
Если вы пропустили эту проблему, просмотрите пример 1.133.

Приготовься 2.10

Упростить: 5−2(n+1)5−2(n+1).
Если вы пропустили эту проблему, просмотрите пример 1.138.

Приготовься 2.11

Умножить: 3(7y+9)3(7y+9).
Если вы пропустили эту проблему, просмотрите пример 1.132.

Приготовься 2.12

Умножить: (2,5)(6,4)(2,5)(6,4).
Если вы пропустили эту проблему, просмотрите пример 1.97.

Решение уравнений с использованием общей стратегии

До сих пор мы имели дело с решением одной конкретной формы линейного уравнения. Пришло время изложить одну общую стратегию, которую можно использовать для решения любого линейного уравнения. Для решения некоторых уравнений, которые мы решаем, не потребуются все эти шаги, но для многих потребуется.

Если начать с упрощения каждой части уравнения, остальные шаги станут проще.

Пример 2,37

Как решать линейные уравнения с помощью общей стратегии

Решите: −6(x+3)=24.−6(x+3)=24.

Решение

Попытайся 2,73

Решить: 5(х+3)=35,5(х+3)=35.

Попытайся 2,74

Решите: 6(y−4)=−18,6(y−4)=−18.

Как

Общая стратегия решения линейных уравнений.
  1. Шаг 1. Максимально упростите каждую часть уравнения.
    Используйте Распределительное свойство, чтобы удалить все скобки.
    Объедините похожие термины.
  2. Шаг 2. Соберите все переменные члены в одной части уравнения.
    Используйте свойство равенства сложения или вычитания.
  3. Шаг 3. Соберите все постоянные члены с другой стороны уравнения.
    Используйте свойство равенства сложения или вычитания.
  4. Шаг 4. Сделать коэффициент переменного члена равным 1.
    Используйте свойство равенства умножения или деления.
    Назовите решение уравнения.
  5. Шаг 5. Проверьте решение. Подставьте решение в исходное уравнение, чтобы убедиться, что результат является верным утверждением.

Пример 2,38

Решить: −(y+9)=8.−(y+9)=8.

Решение
Максимально упростите каждую часть уравнения путем распределения.
Единственный член yy находится в левой части, поэтому все переменные члены находятся в левой части уравнения.
Добавьте 99 к обеим частям, чтобы получить все постоянные члены в правой части уравнения.
Упрощение.
Перепишите -y-y как -1y-1y.
Приведите коэффициент переменного члена к 11, разделив обе части на −1−1.
Упрощение.
Чек:
Пусть y=−17y=−17.

Попытайся 2,75

Решить: −(y+8)=−2.−(y+8)=−2.

Попытайся 2,76

Решите: −(z+4)=−12.−(z+4)=−12.

Пример 2,39

Решите: 5(a-3)+5=-105(a-3)+5=-10.

Решение
Максимально упростите каждую часть уравнения.
Распределить.
Объедините похожие термины.
Единственный член aa находится в левой части, поэтому все переменные члены находятся в одной части уравнения.
Добавьте 1010 к обеим частям, чтобы получить все постоянные члены в другой части уравнения.
Упрощение.
Приведите коэффициент переменного члена к 11, разделив обе части на 55.
Упрощение.
Чек:
Пусть а=0а=0.

Попытайся 2,77

Решить: 2(m−4)+3=−12(m−4)+3=−1.

Попытайся 2,78

Решите: 7(n−3)−8=−157(n−3)−8=−15.

Пример 2,40

Решите: 23(6m−3)=8−m23(6m−3)=8−m.

Решение
Распределить.
Добавьте мм, чтобы получить переменные только слева.
Упрощение.
Добавьте 22, чтобы получить константы только справа.
Упрощение.
Разделить на 55.
Упрощение.
Чек:
Пусть m=2m=2.

Попытайся 2,79

Решите: 13(6u+3)=7−u13(6u+3)=7−u.

Попытайся 2,80

Решите: 23(9x−12)=8+2×23(9x−12)=8+2x.

Пример 2,41

Решите: 8−2(3y+5)=08−2(3y+5)=0.

Решение
Упростить — использовать Распределяющее свойство.
Объедините похожие термины.
Добавьте 22 к обеим сторонам, чтобы собрать константы справа.
Упрощение.
Разделите обе части на −6−6.
Упрощение.
Проверка: Пусть y=−13.y=−13.

Попытайся 2,81

Решите: 12−3(4j+3)=−1712−3(4j+3)=−17.

Попытайся 2,82

Решите: −6−8(k−2)=−10−6−8(k−2)=−10.

Пример 2,42

Решите: 4(x−1)−2=5(2x+3)+64(x−1)−2=5(2x+3)+6.

Решение
Распределить.
Объедините похожие термины.
Вычтите 4x4x, чтобы получить переменные только справа, так как 10>410>4.
Упрощение.
Вычтите 2121, чтобы получить константы слева.
Упрощение.
Разделить на 6.
Упрощение.
Чек:
Пусть x=−92x=−92.

Попытайся 2,83

Решите: 6(p−3)−7=5(4p+3)−126(p−3)−7=5(4p+3)−12.

Попытайся 2,84

Решите: 8(q+1)−5=3(2q−4)−18(q+1)−5=3(2q−4)−1.

Пример 2,43

Решите: 10[3−8(2s−5)]=15(40−5s)10[3−8(2s−5)]=15(40−5s).

Решение
Упрощать сначала от самых внутренних скобок.
Объедините одинаковые термины в скобках.
Распределить.
Добавьте 160s160s, чтобы получить S справа.
Упрощение.
Вычтите 600, чтобы получить константы слева.
Упрощение.
Разделить.
Упрощение.
Чек:
Замените s=−2s=−2.

Попытайся 2,85

Решите: 6[4−2(7y−1)]=8(13−8y)6[4−2(7y−1)]=8(13−8y).

Попытайся 2,86

Решите: 12[1−5(4z−1)]=3(24+11z)12[1−5(4z−1)]=3(24+11z).

Пример 2,44

Решите: 0,36(100n+5)=0,6(30n+15)0,36(100n+5)=0,6(30n+15).

Решение
Распределить.
Вычтите 18n18n, чтобы получить переменные слева.
Упрощение.
Вычтите 1,81,8, чтобы получить константы справа.
Упрощение.
Разделить.
Упрощение.
Чек:
Пусть n=0,4n=0,4.

Попытайся 2,87

Решите: 0,55(100n+8)=0,6(85n+14)0,55(100n+8)=0,6(85n+14).

Попытайся 2,88

Решите: 0,15(40м-120)=0,5(60м+12)0,15(40м-120)=0,5(60м+12).

Классификация уравнений

Рассмотрим уравнение, которое мы решили в начале последнего раздела, 7x+8=-137x+8=-13. Решение, которое мы нашли, было x=−3x=−3. Это означает, что уравнение 7x+8=-137x+8=-13 верно, когда мы заменяем переменную x значением -3-3. Мы показали это, когда проверили решение x=−3x=−3 и вычислили 7x+8=−137x+8=−13 для x=−3x=−3.

Если мы вычислим 7x+87x+8 для другого значения x , левая часть не будет −13−13.

Уравнение 7x+8=-137x+8=-13 верно, когда мы заменяем переменную x значением -3-3, но неверно, когда мы заменяем x любым другим значением. Верно ли уравнение 7x+8=-137x+8=-13, зависит от значения переменной. Подобные уравнения называются условными уравнениями.

Все уравнения, которые мы решили до сих пор, являются условными уравнениями.

Условное уравнение

Уравнение, которое истинно для одного или нескольких значений переменной и ложно для всех остальных значений переменной, является условным уравнением.

Теперь рассмотрим уравнение 2y+6=2(y+3)2y+6=2(y+3). Вы признаете, что левая сторона и правая сторона эквивалентны? Давайте посмотрим, что произойдет, если мы найдем y .

Распределить.
Вычтите 2y2y, чтобы получить yy с одной стороны.
Упрощение — yy исчезли!

Но верно 6=66=6.

Это означает, что уравнение 2y+6=2(y+3)2y+6=2(y+3) верно для любого значения y . Мы говорим, что решением уравнения являются все действительные числа. Уравнение, верное для любого значения такой переменной, называется тождеством.

Личность

Уравнение, верное для любого значения переменной, называется тождеством .

Решением тождества являются все действительные числа.

Что произойдет, если мы решим уравнение 5z=5z−15z=5z−1?

Вычтите 5z5z, чтобы получить одну константу справа.
Упрощение — зз больше нет!

Но 0≠−10≠−1.

Решение уравнения 5z=5z−15z=5z−1 привело к ложному утверждению 0=−10=−1. Уравнение 5z=5z−15z=5z−1 не будет верным для любого значения z. У него нет решения. Уравнение, не имеющее решения или ложное при всех значениях переменной, называется противоречием.

Противоречие

Уравнение, ложное при всех значениях переменной, называется противоречием.

Противоречие не имеет решения.

Пример 2,45

Классифицировать уравнение как условное уравнение, тождество или противоречие. Затем сформулируйте решение.

6(2n−1)+3=2n−8+5(2n+1)6(2n−1)+3=2n−8+5(2n+1)

Решение
Распределить.
Объедините похожие термины.
Вычтите 12n12n, чтобы получить nn на одну сторону.
Упрощение.
Это верное утверждение. Уравнение является тождеством.
Решение — все действительные числа.

Попытайся 2,89

Классифицируйте уравнение как условное уравнение, тождество или противоречие и укажите решение:

4+9(3x−7)=−42x−13+23(3x−2)4+9(3x− 7)=-42x-13+23(3x-2)

Попытайся 2,90

Классифицируйте уравнение как условное уравнение, тождество или противоречие, а затем сформулируйте решение:

8(1−3x)+15(2x+7)=2(x+50)+4(x+3)+18(1−3x)+15(2x+7)=2(x+50) +4(х+3)+1

Пример 2,46

Классифицировать как условное уравнение, тождество или противоречие. Затем сформулируйте решение.

10+4(р-5)=010+4(р-5)=0

Решение
Распределить.
Объедините похожие термины.
Добавьте 1010 к обеим сторонам.
Упрощение.
Разделить.
Упрощение.
Уравнение верно, когда p=52p=52. Это условное уравнение.
Решение: p=52.p=52.

Попытайся 2,91

Классифицируйте уравнение как условное уравнение, тождество или противоречие, а затем сформулируйте решение: 11(q+3)−5=1911(q+3)−5=19

Попытайся 2,92

Классифицируйте уравнение как условное уравнение, тождество или противоречие и укажите решение: 6+14(k−8)=956+14(k−8)=95

Пример 2,47

Классифицировать уравнение как условное уравнение, тождество или противоречие. Затем сформулируйте решение.

5м+3(9+3м)=2(7м-11)5м+3(9+3м)=2(7м-11)

Решение
Распределить.
Объедините похожие термины.
Вычесть 14м14м с обеих сторон.
Упрощение.
Но 27≠−2227≠−22. Уравнение противоречие.
Не имеет решения.

Попытайся 2,93

Классифицируйте уравнение как условное уравнение, тождество или противоречие, а затем сформулируйте решение:

12с+5(5+3с)=3(9с-4)12с+5(5+3с)=3(9с-4)

Попытайся 2,94

Классифицируйте уравнение как условное уравнение, тождество или противоречие и укажите решение:

4(7d+18)=13(3d−2)−11d4(7d+18)=13(3d−2 )−11д

Тип уравнения Что произойдет, если вы ее разгадаете? Решение
Условное уравнение Истина для одного или нескольких значений переменных и ложь для всех остальных значений Одно или несколько значений
Идентификация True для любого значения переменной Все действительные числа
Противоречие Ложь для всех значений переменной Нет решения

Стол 2,5

Раздел 2.

4 Упражнения
Практика ведет к совершенству

Решение уравнений с использованием общей стратегии решения линейных уравнений

В следующих упражнениях решите каждое линейное уравнение.

232.

15(у-9)=-6015(у-9)=-60

233.

21(у-5)=-4221(у-5)=-42

234.

−9(2n+1)=36−9(2n+1)=36

235.

−16(3n+4)=32−16(3n+4)=32

236.

8(22+11р)=08(22+11р)=0

237.

5(8+6п)=05(8+6п)=0

238.

−(w−12)=30−(w−12)=30

239.

−(t−19)=28−(t−19)=28

240.

9(6а+8)+9=819(6а+8)+9=81

241.

8(9b−4)−12=1008(9b−4)−12=100

242.

32+3(г+4)=4132+3(г+4)=41

243.

21+2(м-4)=2521+2(м-4)=25

244.

51+5(4−q)=5651+5(4−q)=56

245.

−6+6(5−k)=15−6+6(5−k)=15

246.

2(9s−6)−62=162(9s−6)−62=16

247.

8(6t−5)−35=−278(6t−5)−35=−27

248.

3(10−2x)+54=03(10−2x)+54=0

249.

−2(11−7x)+54=4−2(11−7x)+54=4

250.

23(9с-3)=2223(9с-3)=22

251.

35(10x−5)=2735(10x−5)=27

252.

15(15с+10)=с+715(15с+10)=с+7

253.

14(20d+12)=d+714(20d+12)=d+7

254.

18−(9г+7)=-1618-(9г+7)=-16

255.

15−(3r+8)=2815−(3r+8)=28

256.

5−(n−1)=195−(n−1)=19

257.

−3−(м−1)=13−3−(м−1)=13

258.

11−4(y−8)=4311−4(y−8)=43

259.

18−2(y−3)=3218−2(y−3)=32

260.

24−8(3v+6)=024−8(3v+6)=0

261.

35−5(2w+8)=−1035−5(2w+8)=−10

262.

4(а-12)=3(а+5)4(а-12)=3(а+5)

263.

−2(a−6)=4(a−3)−2(a−6)=4(a−3)

264.

2(5−u)=−3(2u+6)2(5−u)=−3(2u+6)

265.

5(8−r)=−2(2r−16)5(8−r)=−2(2r−16)

266.

3(4n−1)−2=8n+33(4n−1)−2=8n+3

267.

9(2m−3)−8=4m+79(2m−3)−8=4m+7

268.

12+2(5−3y)=−9(y−1)−212+2(5−3y)=−9(y−1)−2

269.

−15+4(2−5y)=−7(y−4)+4−15+4(2−5y)=−7(y−4)+4

270.

8(х-4)-7х=148(х-4)-7х=14

271.

5(х-4)-4х=145(х-4)-4х=14

272.

5+6(3с-5)=-3+2(8с-1)5+6(3с-5)=-3+2(8с-1)

273.

−12+8(x−5)=−4+3(5x−2)−12+8(x−5)=−4+3(5x−2)

274.

4(u−1)−8=6(3u−2)−74(u−1)−8=6(3u−2)−7

275.

7(2n−5)=8(4n−1)−97(2n−5)=8(4n−1)−9

276.

4(p−4)−(p+7)=5(p−3)4(p−4)−(p+7)=5(p−3)

277.

3(а-2)-(а+6)=4(а-1)3(а-2)-(а+6)=4(а-1)

278.

−(9y+5)−(3y−7)−(9y+5)−(3y−7)
=16−(4y−2)=16−(4y−2)

279.

−(7m+4)−(2m−5)−(7m+4)−(2m−5)
=14−(5m−3)=14−(5m−3)

280.

4[5−8(4c−3)]4[5−8(4c−3)]
=12(1−13c)−8=12(1−13c)−8

281.

5[9−2(6d−1)]5[9−2(6d−1)]
=11(4−10d)−139=11(4−10d)−139

282.

3[−9+8(4ч-3)]3[-9+8(4ч-3)]
=2(5-12ч)-19=2(5-12ч)-19

283.

3[−14+2(15k−6)]3[−14+2(15k−6)]
=8(3−5k)−24=8(3−5k)−24

284.

5[2(m+4)+8(m−7)]5[2(m+4)+8(m−7)]
=2[3(5+m)−(21−3m)]= 2[3(5+м)−(21−3м)]

285.

10[5(n+1)+4(n−1)]10[5(n+1)+4(n−1)]
=11[7(5+n)−(25−3n)]= 11[7(5+n)−(25−3n)]

286.

5(1,2u−4,8)=−125(1,2u−4,8)=−12

287.

4(2,5v−0,6)=7,64(2,5v−0,6)=7,6

288.

0,25(q-6)=0,1(q+18)0,25(q-6)=0,1(q+18)

289.

0,2(р-6)=0,4(р+14)0,2(р-6)=0,4(р+14)

290.

0,2(30n+50)=280,2(30n+50)=28

291.

0,5(16м+34)=-150,5(16м+34)=-15

Классифицировать уравнения

В следующих упражнениях классифицируйте каждое уравнение как условное уравнение, тождество или противоречие, а затем сформулируйте решение.

292.

23z+19=3(5z−9)+8z+4623z+19=3(5z−9)+8z+46

293.

15y+32=2(10y−7)−5y+4615y+32=2(10y−7)−5y+46

294.

5(b−9)+4(3b+9)=6(4b−5)−7b+215(b−9)+4(3b+9)=6(4b−5)−7b+21

295.

9(а-4)+3(2а+5)=7(3а-4)-6а+79(а-4)+3(2а+5)=7(3а-4)-6а+7

296.

18(5j−1)+29=4718(5j−1)+29=47

297.

24(3d−4)+100=5224(3d−4)+100=52

298.

22(3m−4)=8(2m+9)22(3m−4)=8(2m+9)

299.

30(2n−1)=5(10n+8)30(2n−1)=5(10n+8)

300.

7v+42=11(3v+8)−2(13v−1)7v+42=11(3v+8)−2(13v−1)

301.

18u−51=9(4u+5)−6(3u−10)18u−51=9(4u+5)−6(3u−10)

302.

3(6q−9)+7(q+4)=5(6q+8)−5(q+1)3(6q−9)+7(q+4)=5(6q+8)−5( к+1)

303.

5(p+4)+8(2p−1)=9(3p−5)−6(p−2)5(p+4)+8(2p−1)=9(3p−5)−6( р-2)

304.

12(6ч-1)=8(8ч+5)-412(6ч-1)=8(8ч+5)-4

305.

9(4к-7)=11(3к+1)+49(4к-7)=11(3к+1)+4

306.

45(3y−2)=9(15y−6)45(3y−2)=9(15y−6)

307.

60(2x−1)=15(8x+5)60(2x−1)=15(8x+5)

308.

16(6n+15)=48(2n+5)16(6n+15)=48(2n+5)

309.

36(4м+5)=12(12м+15)36(4м+5)=12(12м+15)

310.

9(14д+9)+4д=13(10д+6)+39(14д+9)+4д=13(10д+6)+3

311.

11(8c+5)−8c=2(40c+25)+511(8c+5)−8c=2(40c+25)+5

Математика на каждый день

312.

Ограждение У Мики есть 44 фута ограждения, чтобы собака могла бегать по его двору. Он хочет, чтобы длина была на 2,5 фута больше ширины. Найдите длину L , решив уравнение 2L+2(L-2,5)=442L+2(L-2,5)=44.

313.

Монеты У Ронды есть 1,90 доллара в пятицентовых монетах. Количество десятицентовиков на единицу меньше, чем удвоенное количество пятицентовых монет. Найдите количество пятицентовых монет, n , решив уравнение 0,05n+0,10(2n−1)=1,9.00,05n+0,10(2n-1)=1,90.

Письменные упражнения

314.

Своими словами перечислите шаги общей стратегии решения линейных уравнений.

315.

Объясните, почему следует максимально упростить обе части уравнения, прежде чем собирать переменные члены в одну сторону и постоянные члены в другую.

316.

Какой первый шаг вы сделаете при решении уравнения 3−7(y−4)=383−7(y−4)=38 ? Почему это ваш первый шаг?

317.

Решите уравнение 14(8x+20)=3x−414(8x+20)=3x−4, объяснив все шаги вашего решения, как в примерах в этом разделе.

Самопроверка

ⓐ ​​После выполнения упражнений используйте этот контрольный список, чтобы оценить свое мастерство в выполнении задачи этого раздела.

ⓑ По шкале от 1 до 10, как бы вы оценили свое знание этого раздела в свете ваших ответов на контрольный список? Как вы можете улучшить это?

Темы по алгебре: Введение в задачи Word

Урок 9: Введение в задачи со словами

/ru/алгебра-топики/решающие-уравнения/содержание/

Что такое задачи со словами?

Задача на слов — это математическая задача, записанная в виде рассказа или сценария. По сути, он описывает реальную проблему и просит вас представить, как бы вы решили ее, используя математику. Если вы когда-либо посещали уроки математики, вы, вероятно, решали задачу со словами. Например, это звучит знакомо?

У Джонни есть 12 яблок. Если он отдаст четыре Сьюзи, сколько у него останется?

Вы можете решить эту проблему, взглянув на числа и выяснив, что проблема просит вас сделать. В этом случае вы должны узнать, сколько яблок осталось у Джонни в конце задачи. Читая задачу, вы знаете, что Джонни начинает с 12 яблок. В итоге у него на 4 меньше, потому что он их раздал. Вы можете написать это как:

12 — 4

12 — 4 = 8 , значит, у Джонни осталось 8 яблок.

Речевые задачи по алгебре

Если вы смогли решить эту задачу, вы также должны быть в состоянии решать текстовые задачи по алгебре. Да, они включают в себя более сложную математику, но они используют те же основные навыки решения задач, что и более простые текстовые задачи.

Вы можете решить любую задачу со словами, выполнив следующие пять шагов:

  1. Внимательно прочитайте задачу и выясните, о чем она.
  2. Представляет неизвестных чисел с переменными.
  3. Переведите оставшуюся часть задачи в математическое выражение.
  4. Решить задачу.
  5. Проверьте свою работу.

С помощью этих шагов мы решим задачу по алгебре. Вот типичная проблема:

Стоимость аренды небольшого фургона составляет 30 долларов в день плюс 0,50 доллара за милю. Джада арендовала фургон, чтобы добраться до своего нового дома. Это заняло два дня, и фургон стоил 360 долларов. Сколько миль она проехала?

На первый взгляд это может показаться сложным, но у нас уже есть вся информация, необходимая для ее решения. Давайте рассмотрим это шаг за шагом.

Шаг 1. Внимательно прочитайте задачу.

При возникновении любой проблемы начните с прочтения проблемы. Пока вы читаете, подумайте:

  • Какой вопрос задает задача?
  • Какая информация у вас уже есть?

Давайте еще раз посмотрим на нашу проблему. Какой вопрос задает проблема? Другими словами, что вы пытаетесь выяснить?

Стоимость аренды небольшого фургона составляет 30 долларов США в день плюс 0,50 доллара США за милю. Джада арендовала фургон, чтобы добраться до своего нового дома. Это заняло 2 дня, и фургон стоил 360 долларов. Сколько миль она проехала?

Здесь только один вопрос. Мы пытаемся выяснить , сколько миль Джада проехала . Теперь нам нужно найти любую информацию, которая поможет нам ответить на этот вопрос.

Мы знаем несколько важных вещей, которые помогут нам вычислить общий пробег Джады:

  • Фургон стоил 30 долларов в день.
  • Помимо ежедневных сборов, Jada платила 0,50 долларов за милю.
  • У Джады был фургон 2 дней.
  • Общая стоимость $360 .
Шаг 2: Представление неизвестных чисел с помощью переменных.

В алгебре неизвестные числа представляются буквами, называемыми переменными . (Чтобы узнать больше о переменных, см. наш урок по чтению алгебраических выражений.) Вы можете использовать переменную вместо любой неизвестной вам величины. Глядя на нашу задачу, видите ли вы количество, которое мы должны представить с помощью переменной? Часто это номер, который мы пытаемся узнать.

Стоимость аренды небольшого фургона составляет 30 долларов США в день плюс 0,50 доллара США за милю. Джада арендовала фургон, чтобы добраться до своего нового дома. Это заняло 2 дня, и фургон стоил 360 долларов. Сколько миль она проехала?

Так как мы пытаемся найти общее количество миль, пройденных Джадой , мы будем представлять эту сумму с помощью переменной — по крайней мере, пока мы ее не узнаем. Мы будем использовать переменную м для миль . Конечно, мы могли бы использовать любую переменную, но м должно быть легко запомнить.

Шаг 3: Переведите оставшуюся часть задачи.

Давайте еще раз взглянем на проблему, выделив факты, которые мы будем использовать для ее решения.

Стоимость аренды небольшого фургона составляет 30 долларов США в день плюс 0,50 доллара США за милю . Джада арендовала фургон, чтобы добраться до своего нового дома. На это ушло 2 дня , и фургон стоил 360$ . Сколько миль она проехала?

Мы знаем общую стоимость фургона, и мы знаем, что она включает плату за количество дней плюс еще одну плату за количество миль. Это 30 долларов в день и 0,50 доллара за милю. Проще говоря, это будет:

30 долларов в день плюс 0,50 доллара за милю составляют 360 долларов.

Если вы посмотрите на это предложение и на исходную задачу, вы увидите, что они в основном говорят одно и то же: это стоило Джаде 30 долларов в день и 0,50 доллара за милю, а ее общая стоимость составила 360 долларов. Более короткую версию будет легче перевести в математическое выражение.

Начнем с перевода 30 долларов в день . Чтобы рассчитать стоимость чего-то, что стоит определенную сумму в день, вы должны 90 547 умножить 90 548 дневную стоимость на количество дней — другими словами, 30 в день можно записать как 30 ⋅дней или 30 раз больше количества дней . (Не уверен, почему вы перевели это таким образом? Посмотрите наш урок по написанию алгебраических выражений.)

30 долларов в день и 0,50 доллара за милю составляют 360 долларов

30 долларов ⋅ день + 0,50 доллара ⋅ миля = 360 долларов

Как видите, было несколько других слов, которые мы могли перевести в операторы, поэтому и 0,50 доллара стали + 0,50 доллара, 0,50 доллара за милю стало 0,50 доллара ⋅ миля, а равно стало = .

Далее мы добавим уже известные нам числа и переменные. Мы уже знаем количество дней, в течение которых Джада ездила, 2 , так что мы можем это заменить. Мы также уже сказали, что будем использовать м для представления количества миль, так что мы можем заменить и это. Мы также должны убрать знаки доллара с денежных сумм, чтобы они соответствовали другим числам.

30 $ ⋅ день + 0,50 $ ⋅ миля = 360 $

30 ⋅ 2 + 0,5 ⋅ m = 360

Теперь у нас есть наше выражение. Все, что осталось сделать, это решить ее.

Шаг 4. Решите проблему.

Для решения этой проблемы потребуется несколько шагов. (Если вы не знаете, как выполнять математические действия в этом разделе, возможно, вы захотите просмотреть наш урок по упрощению выражений.) Во-первых, давайте максимально упростим выражение. Мы можем умножить 30 на 2, так что давайте сделаем это. Мы также можем записать 0,5 ⋅ м как 0,5 м .

30 ⋅ 2 + 0,5 ⋅ m = 360

60 + 0,5m = 360

Далее нам нужно сделать все возможное, чтобы получить м отдельно слева от знака равенства. Как только мы это сделаем, мы узнаем, чему равно м — другими словами, это позволит нам узнать количество миль в нашей задаче со словами.

Мы можем начать с избавления от 60 с левой стороны, вычитая его из с обеих сторон .

60 + .5M = 360
-60 -60

44444444444444444444444444444444444444444444444444444444444444444444444444444444444444444444444 4004 4004 40040547 . 5 . Поскольку оно умножается на m , мы сделаем обратное и разделим на него обеих частей уравнения.

.5m = 300
.5 .5

.5 m / .5 is m and 300 / 0,50 равно 600 , поэтому м = 600 . Другими словами, ответ на нашу задачу равен 9.0547 600 — теперь мы знаем, что Джада проехала 600 миль.

Шаг 5. Проверьте проблему.

Чтобы убедиться, что мы решили задачу правильно, мы должны проверить нашу работу. Для этого мы можем использовать только что полученный ответ — 90 547 600 90 548 — и вычислить в обратном порядке, чтобы найти другую величину в нашей задаче. Другими словами, если наш ответ для расстояния Джады верен, мы должны иметь возможность использовать его для работы в обратном направлении и найти другое значение, например, общую стоимость. Давайте еще раз взглянем на проблему.

Стоимость аренды небольшого фургона составляет 30 долларов США в день плюс 0,50 доллара США за милю. Джада арендовала фургон, чтобы добраться до своего нового дома. Это заняло 2 дня, и фургон стоил 360 долларов. Сколько миль она проехала?

Согласно задаче, фургон стоит 30 долларов в день и 0,50 доллара за милю. Если бы Джада действительно проехала 600 миль за 2 дня, она могла бы рассчитать затраты следующим образом:

30 долларов в день и 0,50 доллара за милю

30 ⋅ день + 0,5 ⋅ миля

30 ⋅ 2 + 0,5 ⋅ 600

60 + 300

360

Согласно нашим расчетам, фургон будет стоить 360 долларов, что и указано в задаче. Это означает, что наше решение было правильным. Были сделаны!

Хотя некоторые задачи со словами будут сложнее других, вы можете использовать эти основные шаги для решения любой задачи со словами. На следующей странице вы можете попробовать сами.

Практика!

Давайте потренируемся еще на паре задач. Вы можете решить эти проблемы так же, как мы решили первую — просто следуйте шагам решения проблем, которые мы рассмотрели ранее. Для справки, эти шаги:

  1. Внимательно прочитайте задачу и выясните, о чем она.
  2. Представляет неизвестных чисел с переменными.
  3. Переведите оставшуюся часть задачи в математическое выражение.
  4. Решить задачу.
  5. Проверьте свою работу.

Если вы застряли, вы можете вернуться к задаче на странице 1. Вы также можете посмотреть наш урок по написанию алгебраических выражений, где вы найдете несколько советов по переводу написанных слов в математику.

Задача 1

Попробуйте решить эту задачу самостоятельно. Когда вы закончите, перейдите на следующую страницу, чтобы проверить свой ответ и увидеть объяснение шагов.

Билет на ярмарку в один конец стоит 8 долларов. Семейный проездной стоит на 25 долларов больше, чем половина этой суммы. Сколько стоит семейный проездной?

Задача 2

Вот еще одна задача, которую можно решить самостоятельно. Как и в случае с последней проблемой, вы можете найти ответ и объяснение этой проблемы на следующей странице.

Флор и Мо пожертвовали деньги на одну и ту же благотворительность. Флор пожертвовал в три раза больше, чем Мо. Вместе они пожертвовали 280 долларов. Сколько денег дал Мо?

Задача 1 Ответ

Вот Задача 1:

Билет на ярмарку в один конец стоит 8 долларов. Семейный проездной стоит на 25 долларов больше, чем вдвое. Сколько стоит семейный проездной?

Ответ: $29

Давайте решим эту задачу шаг за шагом. Мы решим ее так же, как решали задачу на странице 1.

Шаг 1: Внимательно прочитайте задачу

Первым делом при решении любой задачи со словами нужно выяснить какой вопрос ставит перед вами задача и определяют информацию, которая поможет вам ее решить . Давайте еще раз посмотрим на проблему. Вопрос тут же на виду:

Билет на ярмарку в один конец стоит 8 долларов. Семейный проездной стоит на 25 долларов больше, чем вдвое. Сколько стоит семейный проездной?

Итак, информация, которая нам понадобится, чтобы ответить на вопрос:

  • Билет в один конец стоит 8 долларов.
  • Семейный проездной стоит на 25 долларов больше, чем половина цена разового билета.
Шаг 2: Представление неизвестных чисел с помощью переменных

Неизвестное число в этой задаче — это стоимость семейного пропуска . Мы представим его переменной f .

Шаг 3: Переведите остальную часть задачи

Давайте еще раз посмотрим на задачу. На этот раз выделены важные факты.

Билет на ярмарку в один конец стоит $8 . Семейный проездной стоит $25, что вдвое больше, чем 9.2524 . Сколько стоит семейный проездной?

Другими словами, можно сказать, что стоимость семейного проездного билета равна половине 8 долларов плюс 25 долларов . Чтобы превратить это в проблему, которую мы можем решить, нам нужно перевести ее в математику. Вот как:

  1. Сначала заменим стоимость семейного пропуска на нашу переменную f .
  2. f равно половине 8 долларов плюс 25 долларов

  3. Затем уберите знак доллара и замените такие слова, как плюс и равно с операторами.
  4. f = половина от 8 + 25

  5. Наконец, переведите оставшуюся часть задачи. Половину числа можно записать как 1/2 раза, или 1/2 ⋅ :
  6. f = 1/2 ⋅ 8 + 25

Шаг 4: Решить задачу

Теперь все, что нам нужно сделать, это решить нашу проблема. Как и с любой проблемой, мы можем решить эту, следуя порядку действий.

  1. f уже один в левой части уравнения, поэтому все, что нам нужно сделать, это вычислить правую часть.
  2. f = 1/2 ⋅ 8 + 25

  3. Сначала умножьте 1/2 на 8. 1/2 ⋅ 8 равно 4.
  4. f = 4 + 25

  5. Затем прибавьте 4 и 25. 4 + 25 равно 29 .
  6. f = 29

Вот и все! f равно 29. Другими словами, стоимость семейного пропуска составляет 29 долларов.

Шаг 5: Проверьте свою работу

Наконец, давайте проверим нашу работу, работая в обратном направлении от нашего ответа. В этом случае мы должны иметь возможность правильно рассчитать стоимость разового билета, используя стоимость, которую мы рассчитали для семейного проездного. Давайте еще раз посмотрим на исходную проблему.

Билет на ярмарку в один конец стоит 8 долларов. Семейный проездной стоит на 25 долларов больше, чем вдвое. Сколько стоит семейный проездной?

Мы подсчитали, что семейный проездной стоит 29 долларов. Наша задача говорит, что пропуск стоит 90 547 $25 больше, чем 90 548, чем 90 547, половина 90 548 стоимости одного билета. Другими словами, половина стоимости одного билета составит 25 долларов меньше , чем 29 долларов.

  1. Мы можем перевести это в следующее уравнение, где s означает стоимость одного билета.
  2. 1/2s = 29 — 25

  3. Давайте сначала поработаем с правой стороной. 29 — 25 равно 4.
  4. 1/2s = 4

  5. Чтобы найти значение s , мы должны найти его в левой части уравнения. Это означает избавление от 1/2. Для этого мы умножим каждую сторону на , обратное от 1/2:2.
  6. с = 8

29 долларов, билет в один конец будет стоить 8 долларов. Глядя на нашу исходную проблему, это правильно!

Билет на ярмарку в один конец стоит 8 долларов. Семейный проездной стоит на 25 долларов больше, чем вдвое. Сколько стоит семейный проездной?

Итак, теперь мы уверены в ответе на нашу проблему: стоимость семейного проездного билета составляет 29 долларов.

Задача 2 Ответ

Вот задача 2:

Флор и Мо пожертвовали деньги на одну и ту же благотворительность. Флор пожертвовал в три раза больше, чем Мо. Вместе они пожертвовали 280 долларов. Сколько денег дал Мо?

Ответ: $70

Давайте рассмотрим эту проблему шаг за шагом.

Шаг 1. Внимательно прочитайте задачу

Начните с вопроса , какой вопрос ставит перед вами задача, чтобы решить ее , и определите информацию, которая поможет вам ее решить . В чем здесь вопрос?

Флор и Мо пожертвовали деньги на одну и ту же благотворительность. Флор пожертвовал в три раза больше, чем Мо. Вместе они пожертвовали 280 долларов. Сколько денег дал Мо?

Чтобы решить задачу, вам нужно узнать, сколько денег Мо пожертвовал на благотворительность. Вся важная информация, которая вам нужна, содержится в задаче:

  • Сумма, пожертвованная Флор, в три раза больше сумма, пожертвованная Мо
  • Сумма пожертвований Флор и Мо составляет Всего 280 долларов
Шаг 2: Представьте неизвестные числа с переменными

Неизвестное число, которое мы пытаемся определить в этой задаче, это Пожертвование Мо . Мы представим его переменной m .

Шаг 3: Переведите оставшуюся часть задачи

Вот снова проблема. На этот раз выделены важные факты.

Флор и Мо пожертвовали деньги на одну и ту же благотворительность. Флор дал в три раза больше, чем Мо . На двоих из них они пожертвовали 280 долларов . Сколько денег дал Мо?

Важные факты проблемы можно также выразить следующим образом:

Пожертвование Мо плюс пожертвование Флор равно 280 долларов

Поскольку мы знаем, что пожертвование Флор составляет , в три раза превышающее пожертвование Мо, мы можем пойти еще дальше и сказать:

Пожертвование Мо плюс утроенное пожертвование Мо равняется 280 долларов

Мы можем перевести это в математическую задачу всего за несколько шагов. Вот как:

  1. Поскольку мы уже говорили, что будем представлять сумму пожертвования Мо переменной m , давайте начнем с замены пожертвования Мо на m .
  2. m плюс три умноженных на m равно $280

  3. Далее, мы можем вставить математических операторов вместо определенных слов. Мы также уберем знак доллара.
  4. m + трижды m = 280

  5. Наконец, давайте математически запишем трижды . Трижды м также можно записать как 3 м или просто 3 м .
  6. м + 3 м = 280

Шаг 4: Решение задачи

Для решения этой задачи потребуется всего несколько шагов.

  1. Чтобы получить правильный ответ, нам нужно получить м только в одной части уравнения.
  2. м + 3м = 280

  3. Для начала добавим м и 3 м . Это 4 м .
  4. 4m = 280

  5. Мы можем избавиться от 4 рядом с m , разделив с обеих сторон на 4.
  6. м = 70

Получили ответ: м = 70 . Другими словами, Мо пожертвовал 70 долларов .

Шаг 5: Проверьте свою работу

Ответ на нашу задачу: $70 , но мы должны проверить, чтобы быть уверенным. Давайте еще раз посмотрим на нашу проблему.

Флор и Мо пожертвовали деньги на одну и ту же благотворительность. Флор пожертвовал в три раза больше, чем Мо. Вместе они пожертвовали 280 долларов. Сколько денег дал Мо?

Если наш ответ правильный, 70 долларов и три раза по 70 долларов должен составить до 280 долларов.

  1. Мы можем написать наше новое уравнение следующим образом:
  2. 70 + 3 ⋅ 70 = 280

  3. Порядок операций требует, чтобы мы сначала умножали. 3 ⋅ 70 is 210.
  4. 70 + 210 = 280

  5. The last step is to add 70 and 210. 70 plus 210 equals 280.
  6. 280 = 280

280 is the combined cost билетов в нашей исходной задаче. Наш ответ правильный : Мо пожертвовал 70 долларов на благотворительность.

Продолжать

Предыдущий: Решение уравнений

Следующая статья:Задачи на расстояние

/en/atomic-topics/distance-word-problems/content/

Учебное пособие по пошаговому решению уравнения с примерами

Утверждение, в котором значение двух математических выражений равно, является уравнением и обозначается знаком равенства знак равно Поэтому обычно мы сталкиваемся с трудностями при решении уравнений и часто задаем как решать уравнения. Если вы тоже боретесь с тем же, статья научит вас методам решения уравнения.

Типы уравнения в математике:-

СОДЕРЖАНИЕ

  1. Линейное уравнение
  2. Уравнение квадратичности
  3. Уравнение 9000. Уравнение 9000. Уравнение 9000. Уравнение 9000. Уравнение 9000. Уравнение 9000. Уравнение 9000. Уравнение 9000. Уравнение 9000. Уравнение 9000. Уравнение 9000. Уравнение 9000. Уравнение 9000. Уравнение 9000. Уравнение 9000. Уравнение 9000. Уравнение 9000. имеют такой тип Y=ax+b, где ‘a’ и ‘b’ — числа, а ‘x’ не может быть равен нулю.

    Квадратное уравнение

    Это уравнения такого типа, где одна переменная из всех содержит показатель степени 2. ax²+bx+c=0 — это квадратное уравнение, где x не равен нулю.

    Радикальное уравнение

    Эти уравнения относятся к такому типу, в котором максимальный показатель степени переменной равен ½, и имеют более одного члена. Обычно переменные в радикальном уравнении лежат внутри радикального символа квадратного корня.

    Тригонометрическое уравнение

    Это уравнения такого типа, в которых на переменные влияют тригонометрические функции.

    Полиномиальное уравнение

    Эти уравнения относятся к такому типу уравнений, в которых вычитается предел старшей степени. В таком уравнении иксы — это все числа, а уравнение состоит из нескольких членов.

    Экспоненциальное уравнение

    Эти уравнения относятся к таким типам, которые содержат переменные вместо показателей.

    Как решить уравнение?

    Процесс решения уравнения:

    Решение уравнения с одной переменной:

    Шаг 1: Написание задачи на решение двухэтапного алгебраического уравнения, чтобы визуализировать решения.

    Шаг 2: Следующим шагом является решение о том, следует ли использовать сложение или вычитание, чтобы изолировать переменные термины. Если одну часть уравнения сложить или вычесть, это нужно сделать с другой частью, чтобы сохранить баланс.

    Шаг 3: В обеих частях уравнения добавление или вычитание константы в обеих частях уравнения, что завершает процесс выделения переменного члена.

    Шаг 4: Коэффициент переменной необходимо исключить путем деления или умножения.

    Шаг 5: Разделив левую часть уравнения, найдите переменные.

    Как решить уравнение с одной переменной с каждой стороны?

    Решение уравнения, имеющего по одной переменной с каждой стороны:

    Шаг 1: Написание задачи для решения гарантирует, что обе переменные одинаковы.

    Шаг 2: Константа перемещается в правую часть уравнения. С помощью сложения или вычитания константа удаляется из левой части уравнения.

    Шаг 3: Переменные перемещаются в левую часть уравнения простым сложением или вычитанием.

    Шаг 4: Разделив обе части уравнения, чтобы выделить переменную, решите переменную.

    Как решить двухшаговое уравнение?

    Однако есть и другие способы решения двухэтапного уравнения:

    можно решить, сохраняя переменную в правой части двухэтапного уравнения. Ответы будут одинаковыми, пока переменные изолированы.

    Умножение в конце вместо деления двухшаговых уравнений может быть решено. Этот тип уравнения решается с помощью арифметики для объединения констант, выделения переменного члена, а затем выделения переменной без члена.

    Как решить уравнение?

    Процесс решения линейного уравнения:

    Шаг 1: При необходимости упростите каждую часть уравнения.

    Шаг 2: Чтобы переместить переменные термины в одну сторону, а все остальные термины в другую, используйте сложение или вычитание.

    Шаг 3: Чтобы удалить все значения, которые находятся перед переменной, используйте умножение или деление.

    Шаг 4: Последним шагом будет проверка ответа.

    Как решить уравнение?

    Процесс решения квадратного уравнения:

    ax²+bx+c=0 — это квадратное уравнение, в котором x не равно нулю.

    Шаг 1: Все члены должны быть разделены на a (коэффициент x²)

    Шаг 2: Числовой член, т. е. (c/a), должен быть перемещен в правую часть уравнения.

    Шаг 3: необходимо заполнить квадрат в левой части уравнения и добавить то же значение в правую часть уравнения, чтобы сбалансировать уравнение.

    Шаг 4: Из обеих частей уравнения извлекается квадратный корень.

    Шаг 5:Число, оставшееся в левой части уравнения, следует вычесть, чтобы найти x.

    Как решить уравнение?

    Процесс решения радикального уравнения:

    Уравнение с одним радикалом:

    Шаг 1: Переменная и радикал изолированы на одной стороне уравнений. Это делается путем объединения одинаковых терминов и добавления или вычитания чисел, чтобы переменная и радикал стояли отдельно друг от друга.

    Шаг 2: Обе части уравнения возводятся в квадрат, чтобы удалить радикалы. Это делается потому, что уравнение должно оставаться сбалансированным.

    Шаг 3: Ответ проверяется в исходных задачах, чтобы убедиться, что ответ реален. Чтобы проверить ответ, просто подставьте каждый ответ вместо «x» в исходном уравнении.

    Как решить уравнение с несколькими радикалами?

    Уравнение с несколькими радикалами:

    Шаг 1: Чтобы получить переменные сами по себе, удалите все радикалы за раз и решите оставшиеся уравнения.

    Шаг 2: Одна из переменных выделяется под радикалами.

    Шаг 3: возведите в квадрат обе части уравнения, чтобы удалить радикал слева.

    Шаг 4: Другой квадратный корень также необходимо выделить.

    Шаг 5: Обе стороны возведены в квадрат, чтобы просто отменить радикалы.

    Шаг 6: Как только все радикалы исчезнут, найдите «x», используя навыки алгебры.

    Шаг 7: Чтобы получить правильный ответ, проверьте все возможные решения.

    Заключение

    При решении уравнения помните, что уравнение является балансом со знаком «=».

    Если что-то делается с одной частью уравнения, то то же самое должно быть сделано и с другой стороной, чтобы уравнение оставалось сбалансированным.

    Просто начните с упрощения каждой части уравнения и после использования сложения или вычитания переместите каждую часть уравнения, содержащую переменную, в одну сторону и из постоянной части изолируйте переменные и, наконец, проверьте решения, вернув исходное уравнение.

Добавить комментарий

Ваш адрес email не будет опубликован. Обязательные поля помечены *